[ /tv/ /rf/ /vg/ /a/ /b/ /u/ /bo/ /fur/ /to/ /dt/ /cp/ /oe/ /bg/ /ve/ /r/ /mad/ /d/ /mu/ /cr/ /di/ /sw/ /hr/ /wh/ /lor/ /s/ /hau/ /slow/ /gf/ /vn/ /w/ /ma/ /azu/ /wn/ ] [ Main | Settings | Bookmarks | Music Player ]

No.93583 Reply
File: Owned_by_Math_by_Tazi_san.jpg
Jpg, 107.99 KB, 693×751 - Click the image to expand
edit Find source with google Find source with iqdb
Owned_by_Math_by_Tazi_san.jpg
Наша кафедра упорно продолжает свою работу. Здесь мы обсуждаем математику, математиков и математические университеты, а также отвечаем на вопросы доброанонов.

Приходящие за советами аноны! Подробно пишите, что именно вы желаете изучать, с какой целью, что уже известно, что не получается.

Следующий, кто спросит, почему 0.(9) = 1, будет покусан.

Предыдущий: >>90593
>> No.93591 Reply
File: cirno_math_class_by_isakysaku-d4sufet.png
Png, 155.09 KB, 781×1023 - Click the image to expand
edit Find source with google Find source with iqdb
cirno_math_class_by_isakysaku-d4sufet.png
>>93569
> Расскажите, а какие последние открытия в математической области?
Посмотри список филдсовских лауреатов. ruwiki://Филдсовская_премия
Например, пятнадцать лет назад Концевич открыл интеграл Концевича, запилил квантовую математику и ввёл в рассмотрение новый математический объект, так называемые "грани" - это сущность, которая возникает при отображении графов и переводит дуги в дуги.
Ещё, насколько знаю, Джет Неструев что-то интересное делает.
> Кстати, реально ли заниматься математикой в каких-либо интересных областях, если твои познания в целом не так то велики
Уже осиливаешь программу вербита? http://imperium.lenin.ru/~verbit/MATH/programma.html
>> No.93593 Reply
Вот вам цитата Бурбаки:
> Математики, кажется, сходятся на том, что между нашими "интуитивными" представлениями о множествах и числах и призванными их описывать формализмами имеется не более чем поверхностное сходство. Разногласия относятся лишь к вопросу о выборе между теми и другими.
Сама я формального определения множества не помню. Впрочем, я глупа и невежественна. ._.
В Москве логика какая-то скучная. Хочу, чтобы рассказывали про топосы, гомотопическую теорию типов и прочие околологические няшности.
>> No.93593 Reply
Вот вам цитата Бурбаки:
> Математики, кажется, сходятся на том, что между нашими "интуитивными" представлениями о множествах и числах и призванными их описывать формализмами имеется не более чем поверхностное сходство. Разногласия относятся лишь к вопросу о выборе между теми и другими.
Сама я формального определения множества не помню. Впрочем, я глупа и невежественна. ._.
В Москве логика какая-то скучная. Хочу, чтобы рассказывали про топосы, гомотопическую теорию типов и прочие околологические няшности.
>> No.93596 Reply
>>93591
Вот книжка. Доставило.
http://yadi.sk/d/3yYqhIlF37AYc
>> No.93607 Reply
Какие по-вашему предметы должен изучать математик? На какие специализации/кафедры она должна делиться?
>> No.93609 Reply
Подскажите пожалуйста, где можно найти доказательство теоремы Александрова о дважды дифференцированности выпуклой функции? Желательно несколько книг, ибо без компиляции редко получается серьезную теорему понять.
Про Эванса-Гариепи знаю, нужно еще.
>> No.93611 Reply
File: tyrande_whisperwind-high-priestess.jpg
Jpg, 313.06 KB, 406×756 - Click the image to expand
edit Find source with google Find source with iqdb
tyrande_whisperwind-high-priestess.jpg
Мне для теории моделей нужно одно понятие. Возможно, я вправе именовать это понятие гомоморфизмом.
Я считаю алгебраической системой только любую упорядоченную пару (S, Q), для которой S непусто, и любой элемент q множества Q является отношением между элементами множества S. Функция при этом также считается отношением. S называю носителем, а Qсигнатурой.
Гомоморфизмом, насколько мне известно, называют отбражение f носителя алгебраической системы в носитель алгебраической системы, которое выявляет подобие систем. При этом для любого отношения q первой системы есть соответствующее отношение q' второй системы, и принадлежность кортежа (u, v, ..., w) элементов носителя первой структуры отношению q влечёт принадлежность кортежа (fu, fv, ..., fw) отношению q'. Значит, мы имеем дело также с отображением сигнатуры в сигнатуру. Возможна частичность отображения.
При моём подходе, как мне кажется, важно не то, является ли гомоморфизм отображением носителя в носитель, а то, что гомоморфизм в силу того, что алгебраическая система есть пара, является парой отображений, из которых, к примеру, первое есть гоморфизм в традиционном смысле, а второе - отображение сигнатуры первой системы в сигнатуру второй. Возможны уточнения, что это отображение инъективно или даже биективно.
Надеюсь, кто-то поможет подобрать для такого понятия подходящий термин. Также рассчитываю получить критику.
>> No.93616 Reply
>>93593
а читала ли ты тексты с сайта Воеводского?

>>93591
> Джет Неструев
Джет Неструев это четрые заслуженных пенсионера, вряд ли они что-то активно сейчас делают.
Но учебник анализа у них квадроохуенный, конечно.
>> No.93617 Reply
>>93616
Можно ссылку на сайт?
>> No.93621 Reply
>>93617
домашняя страничка же
http://www.math.ias.edu/~vladimir/Site3/home.html
кроме гомотопических оснований математики, можешь ознакомиться там с мотивными гомологиями :3

Кстати, люди, интересующиеся гомотопической теорией типов, изучают гомотопическую топологию? У Воеводского там постоянно употребляются слова "стягиваемый", "пространство путей" и т.д. Не зная топологии, невозможно понять эту интуицию же.
>> No.93628 Reply
>>93591
Оценил себя на уровень матшкольника. Это очень печально. Спасибо тебе за информацию, анон.
>> No.93648 Reply
>>93616
Нет, планирую летом читать.
>> No.93648 Reply
>>93616
Нет, планирую летом читать.
>> No.93660 Reply
Я подумал, и получилось, что понятие функции, выражаемое в терминах теории множеств, охватывающее нульарные функции, не нужно, так как нульарная функция - это константа, а константой может служить любое множество, а потому любое множество может называться нульарной функцией.
>> No.93661 Reply
>>93660
Не понимат, как ты пришёл к ненужности.
>> No.93662 Reply
>>93607
> математик
Хуй знает. Я здесь только потому, что нет треда о логическом выводе. А там нужны были бы метатеория формальных систем, теория множеств с теорией моделей и алгебра, изучающая формальные системы алгебраических структур и модели этих систем в расширении теории множеств.
>> No.93663 Reply
>>93661
Поскольку любое множество есть нульарная функция, то термин функция применим к любому множеству.
>> No.93664 Reply
>>93663
Ну и что?
>> No.93665 Reply
>>93664
Тгда слово функция не несёт никакой информации.
>> No.93666 Reply
>>93665
С точки зрения теории множеств и Бурбаки, вообще любой объект - это множество. Функции называют функциями не потому, что они чем-то принципиально отличаются от множеств.
>> No.93698 Reply
Я спрашивал о гомоморфизме и функции. Сейчас я подумал и решил сделать вот как.
Определить упорядоченную единицу:
S тогда и только тогда называется (u) (упорядоченной единицей с первым членом u), когда S=u.
Для n≥1 для получения определения упорядоченной n+1-ки с помощью понятия упорядоченной n-ки использовать такую схему:
S тогда и только тогда называется (u, M) (упорядоченной n+1-кой с первым членом u и с i+1-м членом как i-м членом M), когда M есть упорядоченная n-ка и S={u, {u, M}}. (Можно и S={{u}, {u, M}}, но более глубоких задумок, чем минималистичность, пока нет.)
Далее определить n-арное отношение p (предикат) в S как множество упорядоченных n-к для некоторого n с членами в S. (Тут нужно снова счётное множество определений, так как запись "определения отношения отвлечённо от арности" не умещается в выражение, так как требует счётного множества мест, занятых символами.)
В таком случае предикат p арности 0 в множестве S является множеством только упорядоченных единиц с членами в S, то есть подмножеством S.
Функцию арности не менее 1 можно определить через отношение известным образом: такое отношение, для которого принадлежность ему пар (u, v) и (u, w) влечёт v=w. Но нужно также определить функцию арности 0. Вообще, эвристически ясно, что функцией арности k можно считать специального вида (именно, "однозначное") отношение арности k+1. Как мне подсказывают определение отношения арности 1 и соображение "однозначности отношения", называть функцией арности 0 следует только любое одноэлементное множество (а не вообще, как я писал вчера, любое множество, чтобы понятие функции теряло значимость).
При всём этом для меня вполне удовлетворительно не настолько раздельное определение функций арности 0 и арности строго более 0, а получение определений функций любой арности из одной схемы. Буду рад, если кто-либо поможет мне в поиске такого решения.
>>93666
Конечно, любая функция с точки зрения теории множеств является множеством, но я обращал внимание, что при отвергнутом теперь подходе любое множество является функцией.
>> No.93717 Reply
File: justin-putin.jpg
Jpg, 39.26 KB, 459×861 - Click the image to expand
edit Find source with google Find source with iqdb
justin-putin.jpg
Реквестирую книгу по интегральному исчислению . Нужно понимание интеграла , дифференциал понимаю , интеграл нет . Багаж знаний - школьная программа.
>> No.93720 Reply
File: Todd.jpg
Jpg, 6.27 KB, 225×225 - Click the image to expand
edit Find source with google Find source with iqdb
Todd.jpg
>> No.93722 Reply
>>93720 барели нефти уже тукут к тебе
>> No.93832 Reply
Ви почто катедру утопили? Бумп!
>> No.93836 Reply
А существует ли аналитическое решение задачи по расшифровке трипкодов?
>> No.93836 Reply
А существует ли аналитическое решение задачи по расшифровке трипкодов?
>> No.93848 Reply
>>93836
Формально, да, ибо тупой перебор хешей же.
>> No.93864 Reply
Вышвырнули короче из вуза, и даже не за диффуры или мат.логику, а из-за банального матана, такие дела.
>> No.93866 Reply
>>93864
Теперь у тебя появилось время на то чтобы заняться настоящим матаном. Ура.
>> No.93869 Reply
File: 13112987274.png
Png, 40.30 KB, 163×169 - Click the image to expand
edit Find source with google Find source with iqdb
13112987274.png
>>93864
А у меня завтра с комиссией пересдача. Готов хуево. В армию годен. Ничтожество. Такие дела.
>> No.93883 Reply
>>93864
>>93869
В НМУ почти всем рады.
Как и в армии.
>> No.93884 Reply
>>93583
0.(9) не равен 1, если это не округление. Это же арифметическая истина!
>> No.93886 Reply
>>93884
Давайте подискутируем на эту тему.
Каковы ваши доводы?
>> No.93888 Reply
File: Лангольеры.jpg
Jpg, 50.08 KB, 533×295 - Click the image to expand
edit Find source with google Find source with iqdb
Лангольеры.jpg
>>93884
Я предупреждал - я исполняю. Да будешь ты искусан!
>> No.93889 Reply
Все проблемы с равенством 0.(9) единице, на мой взгляд, проистекают из школьного определения числа. В школе число - это строка символов особого вида, с которой можно делать определённые операции. В строку входят только символы 0,1,2,3,4,5,6,7,8,9, точки и иногда скобочки. Отсюда-то торчат уши у попытки ввести число 0,000...001.
Отсюда, кстати, и постоянные попытки первокурсников выяснить, чему же равно число i - это они таким образом требуют показать им строку арабских цифр, равную i.
Для того, чтобы разрешить псевдопарадокс равенства 0.(9) и 1, школьное определение числа как строки цифр необходимо заменить более строгим определением.
>> No.93891 Reply
File: rPZ6qtoGaX8.jpg
Jpg, 90.87 KB, 427×604 - Click the image to expand
edit Find source with google Find source with iqdb
rPZ6qtoGaX8.jpg
>>93883
Ты мне смерти желаешь что ли.
>> No.93892 Reply
>>93891
> чтоли
фикс
>> No.93893 Reply
>>93892
В русском языке нет слова "чтоли".
>> No.93894 Reply
>>93893
И правда, весь день исправляю ошибки которых не делал. Где тут у вас кафедра психиатрии?
>> No.93896 Reply
>>93889
Скорее из непонимания фундаментальных последовательностей и предела у равнятелей.
Более того, ни в одной публикации и нормальном учебнике нет конвенции равняющей одно другому - а на нет и суда нет.
>> No.93897 Reply
>>93896
0.(9) обозначает предел, к которому сходится фундаментальная последовательность {0.9, 0.99, 0.999, 0.9999, ...}
1 обозначает предел, к которому сходится фундаментальная последовательность {1,1,1,1,...}

Последовательности эквивалентны (по отношению, введённому у Львовского - ссылка в позапрошлом треде), так что пределы равны.

Ещё раз. 0.(9) - это не последовательность, это уже предел.
>> No.93899 Reply
x = 0.(9)
10x = 9.(9)
10x - x = 9.(9) - x
10x - x = 9.(9) - 0.(9)
9x = 9
x = 1
>> No.93900 Reply
>>93899
>>93886
А теперь докажите, что 0.(9) * 10 = 9.(9) и что 9.(9) - 0.(9) = 9.
>> No.93901 Reply
>>93897
> Последовательности эквивалентны
> отношение эквивалентности
Вы в курсе что последнее не даёт нам возможности равнять два объекта не в рамках сколь угодно малого ɛ?
>> No.93903 Reply
>>93901
Говорите конкретнее. Какое эпсилон, что есть равенство, какие два объекта рассматриваем и тэ дэ.
>> No.93904 Reply
Ещё немного о Роскосмосе.
"Я вижу, какие люди приходят сейчас в отрасль. Я беседую с выпускниками. Там знаний на порядок меньше, чем когда мы выпускались. Я до сих пор могу любой интеграл взять, любой! Это долбили в школе и в институте, в академии, вбили", - приводит агентство слова Поповкина.
http://lenta.ru/news/2012/03/30/popovkin/
>> No.93925 Reply
>>93904
Забавно.
>> No.93926 Reply
File: StareHeavy.png
Png, 209.46 KB, 450×330 - Click the image to expand
edit Find source with google Find source with iqdb
StareHeavy.png
>>93904
>>93925
Неужто и неберущийся может?
>> No.93927 Reply
>>93899
Возьмем тождество
-20 = -20
Представим его как 16 — 36 = 25 — 45
Прибавим к обеим частям 81/4
16 — 36 + 81/4 = 25 — 45 + 81/4
В левой части полный квадрат разности чисел 4 и 9/2
В правой части полный квадрат разности чисел 5 и 9/2
(4 — 9/2) в кв = (5 — 9/2) в кв
Извлекаем квадратный корень из обеих частейуравнения.
4 — 9/2 = 5 — 9/2
4 = 5
2 х 2 = 5
>> No.93929 Reply
>>93926
Это ж глава роскосмоса. Он превосходно разбирается в науке и может взять ваш интеграл пятью разными способами.
>> No.93931 Reply
>>93904
Я тоже могу, только он мне до лампочки.
>> No.93932 Reply
>>93904
Взять интеграл каждый дурак может. А ты его счесть попробуй.
>> No.93934 Reply
>>93927
Fail. Модуль потерял.
>> No.93940 Reply
>>93934
2+2=x, х=4
2+2+1=x+1
Корень из (2 + 2 + 1)=Корень из (x+1)
2,2360679775 = корень из (x+1)
2,2360679775 +1,5 = (корень из x+1) + 1,5
3,7360679775 = (корень из x+1) +1,5
3,7360679775 в квадрате = (корень из x+1)в квадрате + 1,5 в квадрате
13,9582039325 = (корень из x+1) + 2,25
13,958203925 - 2,25 = корень из x+1
11,708203925 = корень из x+1
137,082039149 = x + 1
137,082039149 - 1 = х
136,082039149 = х
х = 4
Cледовательно 4 = 136,082039149
>> No.93941 Reply
>>93940
> 3,7360679775 в квадрате = (корень из x+1)в квадрате + 1,5 в квадрате
Ололо, на доске ошибка, я самый сообразительный в своём классе!
>> No.93942 Reply
>>93940
Я предлагаю установить эту тему для разговора традиционно запрещённой на кафедре. Го к модерам. Кто за?
> (корень из x+1) +1,5
> (корень из x+1)в квадрате + 1,5 в квадрате
Я становился таким толстым, что вытекал из бинома Ньютона
>> No.93944 Reply
>>93940
(a+b)^2 = a^2 + 2ab + b^2.
Марш повторять ряд Тейлора.
>> No.93952 Reply
>>93942
Я против. У меня есть отличные лангольеры, ничего более жёсткого не требуется.
>> No.93957 Reply
>>93952
Надо не забыть их упомянуть в ОП-посте!
>> No.93960 Reply
>>93957
А, они и так смутно упомянуты.
>> No.93963 Reply
File: 12723834135037.png
Png, 199.85 KB, 544×400 - Click the image to expand
edit Find source with google Find source with iqdb
12723834135037.png
>>93927
> (4 — 9/2) в кв
> Извлекаем квадратный корень
Низя.
>> No.93964 Reply
>>93963
Можно. Во-первых, операция извлечения квадратного корня из отрицательных вещественных чисел даёт комплексный результат. Во-вторых, там написано "в кв.", то есть он имеет в виду выражение
√((4 − 9/2)²)
>> No.93965 Reply
>>93952
Нам училка в школе на математике для веселья доказывала что 2+2 не равно 4, но я всё забыл, это было давно в девяностых, мы веселились как могли.
>> No.93966 Reply
>>93964
Я знаю. Но формула имеет вид: √x^2=|x|. Он неправильно извлёк, так правильнее сформулировать.
>> No.93967 Reply
>>93966
Yep.
>> No.93971 Reply
Аноны, а что такое категория?
>> No.93988 Reply
>>93971
Спроси у барана который мой ответ удалил.
>> No.93995 Reply
Анон, какой почитать учебник по теорверу и матстату, чтобы взять и все понять? Желательно такой, где мат. ожидание обозначается E, а не M.
>> No.94012 Reply
>>93971
категория Люстерника-Шнирельмана же!
Это минимальное количество стягиваемых замкнутых подмножеств, которыми можно покрыть данное топологическое пространство.
Например, для сферы эта категория равна двум.
Оценки на категорию можно получать с помощью гомологий.
ruwiki://Категория_Люстерника_—_Шнирельмана
>> No.94019 Reply
>>94012
А вдруг он в рамках теории категорий спрашивал?
>> No.94023 Reply
>>94019
Скорее всего так и есть.
>> No.94024 Reply
>>94019
Или про категории Бэра
>> No.94118 Reply
>>94024
А, может, это вообще Аристотелевские категории?
>> No.94120 Reply
>>94118
Являются ли Аристотелевские категории объектами категории всех категорий, вот в чём вопрос
>> No.94121 Reply
>>94120
Да, являются по определению.
>> No.94122 Reply
>>94120
Категории всех категорий категорично являются категориями категорий.
>> No.94150 Reply
Кстати, по-моему таки категории совсем всех категорий таки не существует, или мы рассматриваем категорию всех малых категорий, или сидим в каком-то универсуме.
>> No.94184 Reply
File: наушники-34234.jpeg
Jpeg, 243.84 KB, 811×541
edit Find source with google Find source with iqdb
наушники-34234.jpeg
File: tumblr_mcslskiPDu...
Jpg, 542.95 KB, 1280×1706
Your censorship settings forbid this file.
r-18g
File: anon-592264.jpeg
Jpeg, 29.99 KB, 604×453
Your censorship settings forbid this file.
r-18

Это снова кун, который решил самостоятельно изучить элементарную математику... Помогите, пожалуйста!

((a^(1/3))(x^(1/2)))-((a^(3/4))(x^(1/4)))=a^(1/3)*x^(1/4)(x^(1/4)-a^(5/12))

Вопрос: получилась правая часть? Распишите, пожалуйста.
>> No.94186 Reply
File: IMG_20130315_170429.jpg
Jpg, 129.33 KB, 1024×768 - Click the image to expand
edit Find source with google Find source with iqdb
IMG_20130315_170429.jpg
>>94184
Все показатели степеней - дроби; приводим их к общему знаменателю, это 12.
Потом выносим за скобку a и x в наименьших степенях.
Добра.
>> No.94194 Reply
>>94184
1/2= 1/4 + 1/4
x^(a+b) = x^a * x^b. Когда показатели выражаются друг через друга, обращай на это внимание.
>> No.94196 Reply
>>94186
А как числитель степени 5/12 получился? Я конкретно от этого недоумеваю.
>> No.94197 Reply
>>94196
a^9/12 = a^4/12 * a^5/12
>> No.94199 Reply
>>94186
Не все степени - дроби!
>> No.94200 Reply
>>94199
В этой конкретной задаче - все.
>> No.94212 Reply
>>94197
И правда, лол. Да, математика действительно является наукой, которая покажет насколько ты тупой.
>> No.94239 Reply
File: BEcOn-9CQAAmKQW.jpg
Jpg, 69.47 KB, 700×437 - Click the image to expand
edit Find source with google Find source with iqdb
BEcOn-9CQAAmKQW.jpg
Суп, кафедрач, что про "Анализ" Шварца скажешь?
>> No.94241 Reply
>>94239
Великолепная книга, рекомендую истово, яростно.
>> No.94244 Reply
>>94212
> которая покажет насколько ты тупой.
Которая покажет, насколько ты умеешь в математику. Ну или насколько методично подходишь к её изучению.

мимо-утешающий-себя-заваливатель
>> No.94265 Reply
File: 538cbc8545ee.jpg
Jpg, 83.22 KB, 700×394 - Click the image to expand
edit Find source with google Find source with iqdb
538cbc8545ee.jpg
Доброаноны, порекомендуйте мне пожалуйста книжку, ломящуюся от всяких интересных и содержательных примеров по теории меры.
>> No.94270 Reply
File: belveder.jpg
Jpg, 100.67 KB, 424×633 - Click the image to expand
edit Find source with google Find source with iqdb
belveder.jpg
>>94265
Богачёв, "Основы теории меры" в двух томах. Там тебе и меры, и квантовые меры, и шашечки, и ехать.
>> No.94272 Reply
Доброчаньки, как найти высоту жидкости h2 в усеченном конусе, зная радиусы основ r1 r2 начального усеченного конуса и его высоту h1 , соответственно и его объем V1
Задолбался уже. Гарантия, что должно быть простое решение.
>> No.94273 Reply
File: MQAA6hfB0GU.jpg
Jpg, 61.57 KB, 427×604 - Click the image to expand
edit Find source with google Find source with iqdb
MQAA6hfB0GU.jpg
>>94272
Ах да, еще мы знаем обьем жидкости в этом усеченном конусе V2
>> No.94329 Reply
>>94272
Представь, как круги пробегают усечённый конус от одного основания к другому. Их радиус r очевидно линейно зависит от высоты h. Двух точек достаточно для однозначного определения r = r(h). Итак подбираем, что r(h) = h/h1*(r2-r1) + r1. То есть просто подобрали линейную функцию r(h) так, чтобы r(0)=r1, а r(h1)=r2. Мы не могли сделать это ошибочно, так как через две точки проходит единственная линейная функция.

Жидкость, понятное дело, образует усечённый конус и r(h) даёт радиус её поверхности от её же высоты. А V2 = hпr^2 = hпr(h)^2. Подставил формулу r(h) и получил квадратное уравнение от переменной h. При решении вероятно будет или два корня и только один положительный или только положительный кратный корень. Неоднозначность не возникнет.

Неужели правда сложно? Какой это класс?
>> No.94330 Reply
Котаны, на этой неделе будет олимпиада по математике. Я первокурснота, и срочно нужно почитать что-то по теории вероятности.
>> No.94332 Reply
>>94329
Я делал таким способом. Разве что уравнение объема V = 2hп(r1^2 + 2r1r2 + r2^2)
И если туда подставлять новый r(h), то выйдет кубическое уравнение.
Должен быть метод побыстрее.
>> No.94335 Reply
File: 1243422361427.jpg
Jpg, 99.06 KB, 505×758
Your censorship settings forbid this file.
unrated
>>94330
> теории вероятности
>> No.94336 Reply
File: 1243420419631.jpg
Jpg, 21.24 KB, 255×293
Your censorship settings forbid this file.
unrated
>>94335
> вероятности
> И
>> No.94337 Reply
>>94332
> Разве что уравнение объема V = 2hп(r1^2 + 2r1r2 + r2^2)
Я для цилиндра написал...
> И если туда подставлять новый r(h), то выйдет кубическое уравнение.
Оу, точно. Оно впрочем всё равно решается аналитически.

Я сегодня какой-то вялый.
>> No.94339 Reply
>>94337
Дело в том, что эта подзадача есть частью другой. И смотря на уровень сложности соседних задач, эта таки выбивается из колеи.
В общем, если предположить, что мы знаем формулу объема, не не умеем решать кубические, то как найти хитрожопый и простой выход с ситуации?
>> No.94359 Reply
>>94339
> Дело в том, что эта подзадача есть частью другой.
Так может её надзадачу можно решить не переходя к ней непосредственно? А какова надзадача?
>> No.94361 Reply
Анон, подскажи, если знаешь.
Суть такова: для спортивного программирования есть ресурсы типа codeforces.com, а аналогичных сайтов, помогающих заниматься математикой на продвинутом уровне, до сих пор ни одного не нашёл. Может, ты встречал что-то подобное в интернетах. "Взять учебник и ботать" не предлагать, уже уши вянут от обилия формул на 1 курсе, учитывая, что умение пользоваться мозгом там практически не требуется. Впрочем, похоже, больше ничего и не остаётся.
>> No.94363 Reply
>>94359
Неа, невозможно.
Этот урезанный конус - часть другой фигуры с цилиндрами, нужно находить высоту в зависимости от объема воды. Да и это не важно на самом деле. Просто, или ее можно решить применяя только формулу объема, или составители загнули/ошиблись.
>> No.94381 Reply
File: 1363534961149.png
Png, 1.14 KB, 300×20 - Click the image to expand
edit Find source with google Find source with iqdb
1363534961149.png
>>94361
Попробуй projecteuler.net
>> No.94463 Reply
>>94381
Благодарю, весьма годный сайт, как оказалось.
>> No.94551 Reply
Доброаноны, а не могли бы вы на пальцах разъяснить суть условных распределений и условного матожидания? Нигде такого не видел что-то.
>> No.94552 Reply
Всем утра, пишет запустил-матан-1-курс-кун.

Вобщем сейчас проходим интегралы (конкретно - двойной интеграл), а я так и не научился находить первообразную от функций, ну, если только не считать табличные. Может чего почитать посоветуете?
>> No.94554 Reply
>>94552
Рудин-Демидович-Фихтенгольц жи.
Алсо трай "Письменный — Конспект по высшей математике".
>> No.94555 Reply
>>94554
Алсо забыл Зорича.
>> No.94634 Reply
Аноны, что думаете об этой пасте?

Давай разберёмся, что собой представляет сессия.
Экзамен всегда выполнял и в приличных университетах всё ещё выполняет не столько контрольные, сколько учебные функции. Экзаменационные вопросы в хороших университетах всегда известны ещё до начала курса. По этим вопросам можно увидеть, чему будут учить и каков будет уровень экзаменатора, то есть по одному только взгляду на вопросы сведущий человек может оценить курс и дать совет, стоит ли на него записываться или же следует пойти к другому преподавателю.
В современных рашковузах экзамен выполняет только контрольно-фильтрующие функции и служит инструментом, с помощью которого факультет отчисляет студентов или лишает их права получать крошечную стипендию. В худших случаях сессия позволяет экзаменатору получить дополнительные деньги либо в виде взятки, либо в виде оплаты счёта за академические часы-пересдачи, что есть та же взятка, только легализованная. Об учебной нагрузке экзамена нельзя и помыслить. О том, чтобы вопросы были заранее известны, речь практически никогда не идёт.
Отдельно стоит рассмотреть качество получаемых и проверяемых знаний. Подавляющее большинство преподавателей некомпетентны. Они не владеют предметом, потому их экзамен вырождается в маразм. Я рассмотрю абсурдность таких экзаменов на примере уважаемой мною математики.
Математическая программа всех ВУЗов в этой стране, за исключением НМУ и Вышки, устарела на столетия. Основным учебником математики на первом курсе является книга Фихтенгольца, которая написана немногим менее сотни лет назад, при этом почти полностью скопирована с некоторых работ Коши и тем самым имеет возраст в две сотни лет. Как вы, должно быть, уже догадались, за двести лет математика существенно продвинулась вперёд, и учебник пора бы заменить, но в этой стране преподаватели дифференциального исчисления живут прошлым и не подозревают о существовании современной математики. В курсах математического анализа не упоминают дифференциальные формы, тем самым лишая студента всякой возможности применить математический анализ к чему бы то ни было современному, ни к кластеризации данных, ни к теории ИИ, ни даже к физике. Всё дифференциальное и интегральное исчисление вырождается в зубрёжку таблицы производных без её вывода и двадцати методов "взятия интеграла", при том, что теория интеграла Римана, которой только и известен в России, зубодробительно нудна и никогда строго не излагается.
Курс алгебры зачем-то разделён надвое - на так называемую линейную алгебру, векторной её не называют почти никогда, и на аналитическую геометрию, хотя вторая должна быть всецело включена в первую, ибо представляет собой лишь её координатное приложение. Студенты могут месяцами учить отличие эллиптической поверхности от гиперболической (как будто это хоть кому-то нужно) и не подозревать о существовании понятия идеала. Да что там, подавляющее большинство преподавателей и сами о нём не знают. Ни о дифференциальной геометрии, которая является языком современной физики, ни об алгебраической геометрии, которая важна всем информатикам, ни даже о банальной общей топологии речь никогда не идёт. Студенты даже слов таких не слышат.
Функциональный анализ, вероятности, статистика - интереснейшие и полезнейшие вещи - вообще превратились во что-то непотребное. Доходит до смешного - студент спрашивает преподавателя "где мне это понадобится?", а преподаватель молчит в ответ.
Чуть более конкретно. Вы знаете, как определяется множество в рашкинских курсах математики? "Первичное неопределяемое понятие", вот как. И какую ж математику можно на этом построить? Математика есть логический вывод новых свойств абстрактных конструкций из уже известных свойств. А какие свойства есть у неопределяемого понятия? Да никаких. Математика в рашковузах превращается в шаманство, бесцельное жонглирование непонятными символами без чётного начала и без чёткого конца. Студентов не учат, как доказывать теоремы и и что такое доказательство. Зачастую сам преподаватель этого не понимает. Приведу пример из учебного пособия одного известного ВУЗа, изданного в 2006 году. "Теорема. Для того, чтобы множество было счётным, необходимо и достаточно, чтобы его можно было представить в виде последовательности, то есть в виде A = {a1, a2, a3, ...}. Доказательство. Если множество A можно представить в виде последовательности, то существует взаимно-однозначное соответствие между A и N." Естественно, такое "доказательство" ничего не стоит, поскольку оно непонятно, не логично, оперирует неформализованными терминами и совершенно не обосновано. Тем не менее, уже несколько поколений студентов зубрят эту теорему и доказывают её преподавателю, написавшему пособие. Такой вот самоподдерживающийся маразм. И со всеми прочими теоремами ситуация точно такая же.
Человек с чистым умом не сможет обучиться математике в рашковузах. Он неизбежно натолкнётся на маразм, и дальше у него будет два варианта - либо забить на предмет и на университет, либо отказаться от использования разума и тупо зазубрить идиотские тексты, а затем слово в слово воспроизвести их на экзамене.
Сессия в этой стране - бредовое, бессмысленное и калечащее нервы явление. Мыслящие люди неспособны учиться маразму и сдавать по нему экзамены. Если человек получает хорошие оценки в российском университете, то он почти всегда дурак.
>> No.94640 Reply
>>94336
Буквоедство какое-то.
>> No.94641 Reply
>>94634
Чё-то ты перегибаешь как-то. Местами. Дифигем у нас был, у "чистиков" и механиков, отдельный предмет же. Как его может не быть. Взятием интегралов у было не очень-то много. Интеграл и мера Лебега были. Разве их может не быть?
>> No.94649 Reply
>>94634
Старина Хеллер? Его блог местами интересен, но в нем много и всякой муйни.
>> No.94650 Reply
>>94649
Нет, не Хеллер. Паста не гуглится, вроде ориджинал.
>> No.94686 Reply
>>94634
Я думаю, что автор прав в общем, но допущено много небольших несоответствий истинному положению дел, касающихся программы курсов, преподаваемых в университетах, оценок современности материала и связи с приложениями.
Не уделяется внимание крайне платонистской методологии, гегемония которой является заслугой школы. И так не только в России.
>> No.94687 Reply
>>94686
Да, мне платонистские взгляды противны. Я приверженность формализма и интуициониста я могу считать коллегой, потому что он критически и внимательно настроен по отношению к логике, но ненавижу беззаботность классического взгляда.
>> No.94689 Reply
>>93583
Здравствуйте, уважаемые аноны. Я хочу попросить у вас достойные учебники по алгебре. Дело в том, что я пребывал достаточно долгое время в апатии. И вот, хочу наверстать все упущенное.
>> No.94699 Reply
>>94689
http://yadi.sk/d/FeBOATVX1ep6m - общепит
http://yadi.sk/d/TVRB4HPb08kDQ - классика жанра
http://yadi.sk/d/TbAYh60q0SMIt - творческая компиляция
>> No.94708 Reply
Добрый вечер, кафедра. Зачем нужны производящие функции?
>> No.94709 Reply
>>94708
Какого вида ответ ты ожидаешь услышать на этот вопрос?
>> No.94717 Reply
>>94699
А что у тебя ещё есть из "не-общепита"? Есть что-либо по математическому программированию, линейному и нелинейному, выпуклому анализу, вариационному исчислению, исследованию операций? А по численным методам? Теорверу?
>> No.94719 Reply
File: iH5t7.jpg
Jpg, 167.76 KB, 988×681 - Click the image to expand
edit Find source with google Find source with iqdb
iH5t7.jpg
>>94709
Ну, например, какие интересные факты доказываются с их использованием.
>> No.94723 Reply
>>94689
Куликов Л., Алгебра и теория чисел
Нечаев В., Числовые системы
Я раньше думал, что Ленг норм, но потом посмотрел, и не понравилось.
>> No.94728 Reply
>>94717
http://www.mccme.ru/ium/old_courses.html
Выбирай курс, смотри список литературы.
>> No.94729 Reply
В комбинаторике:
http://www.mccme.ru/ium/ancient/combs93.html

В теории вероятностей в основном используются для доказательства разных предельных теорем, а также для вычисления моментов.
>> No.94814 Reply
Матемач, помоги.
Пусть у меня есть гауссова величина Y ~N(\mu,\sigma). Какое будет распределение у ее квадрата? У корня (1 + Y^2)?
>> No.94840 Reply
File: Screenshot-from-2013-03-21-12:20:59.png
Png, 28.98 KB, 687×180 - Click the image to expand
edit Find source with google Find source with iqdb
Screenshot-from-2013-03-21-12:20:59.png
>>94814
Disregard that.
Подскажите, как подступиться к ширяевскому пикрелейтеду.
>> No.94845 Reply
>>94840
Можно "в тупую":
Найдём плотность f(x, y, z), где x = искомая величина, y = \xi2, z = \xi3. Замена обратимая, гладкая, так что всё ок.
Теперь, чтобы найти плотность x, проинтегрируем по f(x, y, z) по y, z
Получим плотность нормальной величины
>> No.94850 Reply
>>94840
Придумал, как проще доказать.
x = искомая величина, y = \xi2, z = \xi3
При фиксированном z, очевидно, имеем распределение N(0, 1), т.к.
\xi1 и yz - независимые нормальные => их сумма нормальная, её дисперсия = 1 + z^2
Т.к. условное распределение x не зависит от z, то оно равно маргинальному.
>> No.94852 Reply
File: 40p2yjvsRQQ.jpg
Jpg, 44.45 KB, 459×304 - Click the image to expand
edit Find source with google Find source with iqdb
40p2yjvsRQQ.jpg
>>94850
Ого как круто, спасибо. Поначалу не понимал, но вроде разобрался. У меня теперь только вопрос, каким путем ты пришел к такому-то решению?
>> No.94853 Reply
>>94850
Еще один вопрос задам - верно ли, что от распределения \xi3 здесь вообще ничего не зависит?
>> No.94890 Reply
Мудроаноны, еще один вопрос у меня по вероятностям.
Есть плотность на всем R^n. Как найти условное распределение, если x принадлежит некоторому известному многообразию?
>> No.94925 Reply
Подкину еще задачку в копилку навеки нерешенных, лол.

Как n-мерном пространстве найти расстояние от начала координат до отрезка?
>> No.94931 Reply
>>94925
Методами нелинейного программирования можно решать как-то. Но я не знаю, можно ли не-численно. Это же поиск минимума. Так?
>> No.94955 Reply
File: v401_23.pdf
Pdf, 0.17 KB, 612×792 - Click the image to get file
v401_23.pdf
>>94852
Ну хз, это же упражнение, поэтому сразу ищешь решение в 1-2 действия, отсекая заведомо сложные пути.
>>94853
Используется то, что \xi3 не зависит от (\xi1, \xi2). А так форма распределения в принципе любая.
Где используется: при взятии условного распределения (\xi1, \xi2) | \xi3. Если (\xi1, \xi2) зависит от \xi3, то условное распределение может не быть нормальным.
>>94890
Интегрировать, например, как в рилейтед
>>94925
Разве это не минимум(длина перпендикуляра, расстояние до одного из концов)?
>> No.94959 Reply
>>94955
> длина перпендикуляра
Прекрасно. Осталось дело за малым. Ты ведь нам скажешь, как найти длину перпендикуляра?
> Разве это не минимум(длина перпендикуляра, расстояние до одного из концов)?
Нет, перпендикуляр всегда минимален, но другое дело, что точка прямой, откуда идёт перпендикуляр может не принадлежать прямой. Вот тогда решение - расстояние от ближаёшего к перпендикуляру конца.
>> No.94970 Reply
>>94959
> Прекрасно. Осталось дело за малым. Ты ведь нам скажешь, как найти длину перпендикуляра?
Блин, а ведь это и правда легко. Какой же я тупой.
>> No.95006 Reply
>>94959
> перпендикуляр всегда минимален, но другое дело, что точка прямой, откуда идёт перпендикуляр может не принадлежать прямой. Вот тогда решение - расстояние от ближаёшего к перпендикуляру конца
Да, я это имел в виду, просто написал чепуху
>> No.95012 Reply
>>95006
А между двумя скрещивающимися прямыми кратчайшее - это перпендикуляр к обоим прямым так?
>> No.95014 Reply
>>95012
Да, т.к. кратчайшее от точки до прямой - перпендикуляр.
Т.е. любой неперпендикулярный отрезок всегда можно сделать короче.
>> No.95100 Reply
Почему 0.(9) равно 1?
>> No.95102 Reply
>>95100
Потому что равно. Давайте не выносить это в оп-пост.
>> No.95130 Reply
>>95100
Баги n-ичной системы же.
Вариант: по определению десятичной записи.
>> No.95133 Reply
>>95130
Кстати, как она определяется?
>> No.95137 Reply
>>95133
Для любых натуральных n, a, b, ..., c, где a,b,c<10 n,ab...c взаимно заменяемо n+a/10+b/10^2+...+c/10^k. То же самое я мог записать яснее явно указывая последовательность слагаемых сложной суммы, но без нижних индексов это только затруднит восприятие, так что надеюсь, что понимание записи произойдёт в соответствии с вложенным мной смыслом. Эта запись предполагает натуральные числа начинающимися нулём, если натуральные числа предполагаются начинающимися единицей, то вносится очевидная поправка. И ещё такоей каламбур: суммы, конечно, конечны.
Бесконечные дроби не целесообразны, поскольку не могут входить в такие конечные линейные последовательности как термы или формулы.
>> No.95138 Reply
>>95133
Вопрос предоставил пищу моим размышлениям.
Пожалуй, применение бесконечных дробей к теории непрерывного поля не требует трактовать их как бесконечные линейные последовательности символов алфавита некоторого языка. Основной целью является построение модели непрерывного поля, а БД представляют собой термы теории, допускающей понятия цифры, противоположности элементов и бесконечной последовательности.
>>95137
>> No.95140 Reply
>>95137
Конечно, нецелесообразны в качестве записей термов. Но они же ими не являются, лол.
Определяются как отображения N -> {0,12,3,4,5,6,7,8,9}, и всё. Термами являются штуки из теории множеств или чего-нибудь покруче.
>> No.95141 Reply
Задача из ЗНО 3 уровня:
Через гипотенузу прямоугольного треугольника проходит плоскость. Углы между этой плоскостью и катетами прямоугольного треугольника — альфа и бета. Нужно найти угол между плоскостью, в которой лежит треугольник, и плоскостью, его пересекающей.
Ясно, что здесь получается пирамида SABC с прямоугольным треугольником в основании. При этом SA перпендикулярна плоскости ABC. Углы SCA и SBA — альфа и бета соответственно. Проводим перпендикуляр из точки A на прямую BC, то есть высоту прямоугольного треугольника AH, и получаем угол SHA, который нам и надо найти. Как его найти?
>> No.95143 Reply
>>95140
> Определяются как отображения N -> {0,12,3,4,5,6,7,8,9}
Надо ещё целую часть упомянуть.
> Термами являются штуки из теории множеств или чего-нибудь покруче.
Достаточно языка без теории.
>> No.95144 Reply
>>95143
> Надо ещё целую часть упомянуть.
Ой, спасибо.
> Достаточно языка без теории.
Тащемто, не понял. Язык - примитивный интрумент, теория - навык пользования языком, а все это нужно, чтоб понять, как устроен мир, вот так мне кажется.
Формальные языки (уж, тем более, первого порядка) сами по себе неинтересны и нужны, как сиденья в автомобиле.
>> No.95148 Reply
>>95141
Используй теоремы для трёхгранных углов, там вроде всё легко получается.
>> No.95189 Reply
>>95141
У тебя SBC - данный прямоугольный треугольник, а SA - перпендикуляр на плоскость?
SH=SBSC/BC=SBSC/кор(SB^2+SC^2) (1) кор(...)- квадратный корень из ...
SB=SA/sin(бета) (2)
SC=SA/sin(альфа) (3)
Подставляя (2) и (3) в (1) получаем (после сокращений):
SH=SA/кор(sin^2(альфа)+sin^2(бета))
sin(SHA)=SA/SH=SA*кор(sin^2(альфа)+sin^2(бета))/SA=кор(sin^2(альфа)+sin^2(бета))

Угол SHA=arcsin(кор(sin^2(альфа)+sin^2(бета)))
>> No.95199 Reply
>>95144
> Язык - примитивный интрумент, теория - навык пользования языком, а все это нужно, чтоб понять, как устроен мир, вот так мне кажется.
Допустимый взгляд. Я просто сказал, что для определения термов достаточно языка теории.
>> No.95289 Reply
File: 0_26463_f3234c5c_L.jpg
Jpg, 53.25 KB, 500×484 - Click the image to expand
edit Find source with google Find source with iqdb
0_26463_f3234c5c_L.jpg
Матемач, а почему собственные частоты колебаний называются собственными? Ведь непспроста же.
>> No.95303 Reply
File: Пустое_множество.JPG
Jpg, 11.32 KB, 241×67 - Click the image to expand
edit Find source with google Find source with iqdb
Пустое_множество.JPG
>>93583
Аноны, встречается ли бурбаковское тау у кого-нибудь кроме Бурбаки?
>> No.95321 Reply
>>95303
Нет.
>> No.95326 Reply
File: Avy_18HtIuM.jpg
Jpg, 68.19 KB, 600×450 - Click the image to expand
edit Find source with google Find source with iqdb
Avy_18HtIuM.jpg
>>93583
Анон, объясни мне, как доказали великую теорему Фермы.
>> No.95330 Reply
>> No.95331 Reply
>>95303
Это Гильбертово эпсилон-исчисление, очень популярная в современной матлогике вещь. Связана с лямбда-исчислением.
enwiki://Epsilon_calculus
Бурбаки чисто из эстетических соображений взял тау вместо эпсилон.
>> No.95348 Reply
Анон, не в курсе, где можно найти много примеров решения неопределённых интегралов? Чем больше, тем лучше. Наиболее простые примеры отлично понимаю, а вот со сложными беда.
>> No.95368 Reply
>>95348
В задачнике Демидовича. Прорешай его.
>> No.95370 Reply
>>95368
Спасибо, антидемидович вполне подошёл.
>> No.95607 Reply
File: MQm8DwccHLM.jpg
Jpg, 33.93 KB, 600×491 - Click the image to expand
edit Find source with google Find source with iqdb
MQm8DwccHLM.jpg
>> No.95671 Reply
File: CityInfo_tip.png
Png, 8.47 KB, 415×237 - Click the image to expand
edit Find source with google Find source with iqdb
CityInfo_tip.png
>> No.95713 Reply
>>93583

Может кто-нибудь может объяснить как это работает, и почему?
http://cs521306.vk.me/u16000386/doc/5922a5b05127/file.gif
>> No.95719 Reply
>>95713
Бесконечный шоколад! Бля, спасибо, анон, я твой должник.

нищеброд-кун
>> No.95725 Reply
>>95713
В Белоруссии все так плохо с математикой?
>> No.95726 Reply
File: triangle.gif
Gif, 8.25 KB, 499×516
edit Find source with google Find source with iqdb
triangle.gif
File: 1364602630434.png
Png, 1.17 KB, 300×20
edit Find source with google Find source with iqdb
1364602630434.png

>>95713
Оптический обман и компьютерная коррекция.
>> No.95727 Reply
File: extr_kor.pdf
Pdf, 0.18 KB, 612×792 - Click the image to get file
extr_kor.pdf
>> No.95733 Reply
>>95725

Откуда ты узнал мое местонахождение, чертов шпион?
>> No.95739 Reply
>>95733
Ты сам написал, что ты из Минска, когда заполнял профиль.
>> No.95742 Reply
>>95726
Ага, иллюзии. Фокусы-покусы!
>> No.95760 Reply
>>95739

Какой профиль? >_>
>> No.95770 Reply
File: 1348003631001.jpg
Jpg, 44.27 KB, 500×750 - Click the image to expand
edit Find source with google Find source with iqdb
1348003631001.jpg
>> No.95773 Reply
>>95713
Линия, вдоль которой идёт разрез кажется одним отрезком, но на самом деле это ломаная из двух отрезков и при развороте она меняет ориентацию. А дальше подлая компьютерная корректировка(должна была быть некая четырёхугольная щель.)
>> No.95774 Reply
>>95773
Какие пакеты есть для символьных и численных вычислений и построения графиков под линь по-лучше? Чтоб если что можно было решить СЛАУ, интеграл, уравнение или построить графики.
>> No.95775 Reply
>>95774
сайлаб.
>> No.95789 Reply
Дорогие аноны, осмотревшись среди поля научных дисциплин я понял, что для изучения любой из них мне необходимы знания математики, которых, как и любых других, я не имею.
Математика мне приятна и сама по себе, но подскажите пожалуйста хороший учебник (или серию, но серия выйдет дороже) который бы охватывал весь школьный курс. С задачами для проверки.
Среди всего многообразия нужен компетентный совет для эффективного выбора.
>> No.95790 Reply
>>95789
Скажи, анон, а зачем тебе школьный курс?
>> No.95791 Reply
>>95790
Потому что я его не знаю. Нельзя же (вероятно) начать сразу с чего то большего.
>> No.95792 Reply
>>95791
Но откуда тогда ты знаешь, что ты его не знаешь?
Про что ты хочешь узнать?
> Нельзя же (вероятно) начать сразу с чего то большего
В этом у меня есть некоторые сомнения.
>> No.95793 Reply
>>95789
Давай учить математику вместе!
асечку запили.
>> No.95794 Reply
>>95791
Школьная математика не представляет из себя что-либо целостное, как и большинство других школьных предметов. В каком-то смысле изучение многих предметов в вузах идёт заново. Тебе могут потребоваться только школьная тригонометрия, немного школьной геометрии и свойства степеней и логорифмов. И то фиг его знает.
>> No.95795 Reply
>>95792
> откуда тогда ты знаешь, что ты его не знаешь?
Отсутствие не требует доказывания и определения. Иначе мы скатимся к такому немыслимому абсурду как "Докажи что бога нет".
> этом у меня есть некоторые сомнения.
Могут быть исключения, но, видится мне, они носят персональный характер.
>> No.95797 Reply
>>95795
> Отсутствие не требует доказывания и определения.
В математике полно теорем доказывающих отсутствие чего-то между прочим. Пока не доказано, есть или нет, то оно может быть, а может и не быть. А ещё бывает, что для утверждения доказано, что его невозможно ни доказать ни опровергнуть. Тогда можно взять хоть его, хоть его отрицание в качестве аксиомы.
>> No.95798 Reply
>>95793
644877270

>>95794
> в вузах идёт заново
Мне не нужны знания под ВУЗ, я бы хотел их сами по себе, без иной цели кроме получения удовольствия от них.
>> No.95799 Reply
В общем, аноны, читайте "Анализ" Зорича параллельно с "Курсом высшей математики" Демидовича. Там достойно вводится вся нужная школьная математика. И поглядывайте в википедию по ходу чтения. Определение синуса там, дифференциала и прочего.

Школьные учебники - совершенно зряшная вещь, только время зря потратите.
>> No.95800 Reply
>>95795
> Отсутствие не требует доказывания и определения.
Требует.
Ты можешь уже владеть на школьном уровне всем, что изучается в школах. Умеешь складывать и вычитать дроби, слышал про возведение в степень, знаешь про многочлены и знаком с понятием "решить уравнение" - знаешь школьный курс (за исключением некоторых деталей, которые в школе всё равно изучаются как попало).
А школьная "геометрия" не нужна совсем.
>> No.95802 Reply
>>95799
Спасибо, ты дал мне направление, о котором я не задумывался (точнее отбрасывал такие идеи).
>>95800
> А школьная "геометрия" не нужна совсем.
Но те же формулы площадей часто требуются в других дисциплинах. Вот та же формула пути при равномерном ускорении движения - площадь трапеции.
>> No.95803 Reply
>>95802
Лол. Формула пути, объём цилиндра/конуса, длина/площадь окружности, - это интегралы и ничего более. Площадь треугольника - произведение векторов.
Зубрить всё это отдельно не только не нужно, но и вредно.
>> No.95804 Reply
>>95799
Это и есть школьная, но для обычной высшей школы. Для средней школы уже советовали много в тредах. Выходит по 5 книг. Для изучения, как я прогнозирую, нужно от полугода до года.
>> No.95809 Reply
>>95803
Почему вредно?
>> No.95810 Reply
>>95809
Не способствует отчётливому пониманию.
>> No.95826 Reply
>>95809
Потому что ведёт к отупению. Например, школьников в школах учат, что числа - это всегда строки из 0..9. Потом эти школьники долго-долго допытываются, чему же равно число i.
>> No.95830 Reply
>>95826
Нужна теория полей же. И анализ по топологии.
Вообще, программа сильно перегружена, нужно её упростить бы в двух направлениях: с одной стороны, раза в три сократить по объёму, а с другой — сделать более логичной.
>> No.95832 Reply
>>95830
Кому нужна? Зачем существует программа, какова цель обучения?
>> No.95837 Reply
>>95832
Я считаю, складывать и умножать в рациональном поле нааучились бы, при этом все бы было намного проще.
>> No.95842 Reply
>>95837
Зачем в жизни такой уровень абстракции?
>> No.95866 Reply
>>95842
В жызни такой уровень абстракции совершенно не нужен.
В жызни нужна водочка, селёдочка под шубкой и чтобы в офисе на напрягали.

привет всем в этом чятике
>> No.95868 Reply
>>95866
И в этом нет ничего плохого, это нормальная здоровая жизнь. Не понимаю, почему няши с тифаретника так ненавидят нормальных людей.
>> No.95899 Reply
>>95868
> здоровая жизнь
> водочка
No.
>> No.95901 Reply
>>95899
Yep. Водочка - это продукт питания, внезапно.
>> No.95902 Reply
>>95826
Хм... Я не помню каких-либо попыток определять числа в школе... К тому же в школе пишут букву R и приминяют слово "действительные", так что вроде ясно, что есть и другие. Ерунда какая-то.
>>95832
Разные поля используют в разных целях. Например помимо банальных R и C есть конечные поля, используемые где-то в теории кодирования. При этом полезно называть и то и то одним термином поля, чтобы оди и те же теоремы работали сразу и для того и для того. Абстракция нужна, чтобы одна теорема применялась к разному. Чем выше уровень абстракции, тем теоремы универсальнее.
>> No.95904 Reply
>>95902
> Я не помню каких-либо попыток определять числа в школе
Они определяются как конечные строки символов с аксиоматически заданными операциями сложения и умножения. Таблицу сложения и таблицу умножения всех заставляют учить назубок.
После такого подхода объяснить, что строчка цифр - это лишь модель числа и что, вообще говоря, возможны различные изоморфные друг другу модели, очень сложно.

Похожие проблемы возникают с векторами. Пока бывший школьник отучится считать вектор единственно только направленным отрезком, не один месяц пройдёт.
>> No.95915 Reply
>>95904
> Они определяются как конечные строки символов с аксиоматически заданными операциями сложения и умножения.
> > конечные
Как так? В старших классах школы во все поля работают с иррациональными числами. Ерунда какая-то. Может, это зависит от школы?
> После такого подхода объяснить, что строчка цифр - это лишь модель числа и что, вообще говоря, возможны различные изоморфные друг другу модели, очень сложно.
Сдаётся мне, это вообще всегда сложно даётся студентам. Хотя фиг его. Но в моей школе я не помню ничё такого.
> Похожие проблемы возникают с векторами. Пока бывший школьник отучится считать вектор единственно только направленным отрезком, не один месяц пройдёт.
На школьной физике(или не только на школьной?) любят давать свободные, скользящие и закреплённые вектора, кстати. Я вообще в упор не понимал и щас не понимаю, что такое закреплённые и скользящие и зачем они.
>> No.95919 Reply
>>95915
> Как так?
Числа - иррациональные. А строки символов - конечные. Видел волшебное школьное троеточие, десу?
> Сдаётся мне, это вообще всегда сложно даётся студентам.
Если объяснить суть абстрагирования, то изучение алгебры пройдёт гораздо легче.
>> No.96123 Reply
Я тут упарываю Шварца, "Анализ". Есть задачи конкретно к этой книге?
>> No.96156 Reply
>>95919
> Видел волшебное школьное троеточие, десу?
Не видел.
>> No.96173 Reply
Привет, анон.

Как развить интуицию для ментального жонглирования теоремой Байеса?
>> No.96186 Reply
>>96156
Показываю. "pi = 3.14159265..."
Строка символов - конечная.
>> No.96193 Reply
>>95904
> вектор единственно только направленным отрезком, не один месяц пройдёт.
А на самом деле как?
мимолитературовед
>> No.96204 Reply
Переименуйте этот тред в "кафедру невежества", лол
>> No.96205 Reply
>>96204
С какой стати, няша?
>> No.96218 Reply
Анон, пытаюсь освежить знания по теории вероятеостей, нужны только самые основы, да нагуглить не смог.
Положим, бросаем 6 шестигранных кубиков.
Как рассчитать вероятность того, что хотя бы на одном из них выпадет шестёрка?
Вероятность того, что на каждом из них выпадет шестёрка равна 1/46656?
>> No.96219 Reply
>>96218
> Как рассчитать вероятность того, что хотя бы на одном из них выпадет шестёрка?
Это 1- вероятность того, что не выпала ни одной шестёрка. 1-(5/6)^6
На каждом - да, (1/6)^6
>> No.96226 Reply
>>96205
Чтобы название соответствовало содержанию.
Тут всегда было своеобразно, но вот этот >>96186 гражданин уже ни в какие ворота.
>> No.96227 Reply
>>96226
Лол. Формализуй это троеточие.
>> No.96228 Reply
>>96226
Ну главный элемент невежства в этом треда - это всегда были неймфаги (ты и ещё один, предшественник который тоже ты).
>> No.96229 Reply
>>96227
> Формализуй это троеточие
Здравствуйте. Вы баснословно тупой.
>> No.96230 Reply
>>96229
Привет. Ты пень, который не въезжает в суть дискуссии.
>> No.96231 Reply
File: 125666883213859.gif
Gif, 1101.85 KB, 256×192
Your censorship settings forbid this file.
unrated
>>96230
Суть дискуссии в том, что нельзя формализовать любую глупость, которая родится в твоей пустой голове.
Есть определения, которые нужно знать. То, что в школе не определяются действительные числа, это факт, да. Там вообще ничего не определяется. Но это не повод заменять одну глупость другой.
>> No.96234 Reply
>>96193
Вектор, уважаемый литературовед, это элемент векторного пространства, абстракция того же сорта, что и числа. Как натуральные числа ты можешь смоделировать камешками, так и векторы ты можешь смоделировать направленными отрезками. Но из возможности моделирования не следует, что числа есть камешки, а векторы - направленные отрезки. Натуральные числа можно смоделировать не только камешками, векторы можно смоделировать не только направленными отрезками.
>> No.96235 Reply
File: oblivaniya.jpg
Jpg, 211.76 KB, 700×467 - Click the image to expand
edit Find source with google Find source with iqdb
oblivaniya.jpg
>>96231
Ты - тупой чванливый сосачер, оскорбляющий честных анонов. Ты даже не попытался прочитать тред, просто выхватил из контекста фразу и сагрился на неё.
>> No.96237 Reply
>>96235
Вот это новости! Ты ещё будешь стоить относительно меня фантазии гипотезы? в таком случае сразу иди нахуй
Я отвечал тому гражданину, который посоветовал мне "формализовать троеточие".
Если ты тот, кто тонко иронизировал над школьными "определениями", считая что я не понял твоей "тонкоты", то просто проходи мимо, не задерживайся.
>> No.96241 Reply
я сегодня не очень бобр
>> No.96243 Reply
File: a109408_angel30.jpg
Jpg, 58.53 KB, 400×500 - Click the image to expand
edit Find source with google Find source with iqdb
a109408_angel30.jpg
>>96241
Тогда добра тебе. :3
>> No.96255 Reply
>>96173
Удваиваю вопрос. Шучу, это я же.
>> No.96257 Reply
>>96173
>>96255
Никак, если ты пошел в математики, то с интуицией у тебя, скорее всего, очень плохо.
>> No.96258 Reply
>>96257
Агмх.
>> No.96299 Reply
Анон, а что, теория вероятностей уже не в моде?

В интернете, на курсере, opencourseware тысяча курсов по статистике, теорверу и статистике, но нигде нет нормального последовательного курса по теории вероятностей.
>> No.96360 Reply
>>96299
По идее, фундамент теорвера изучается в курсе анализа и теории меры.
Всё остальное - это приложения теорвера, т.е. как раз статистика, стохастические процессы и т.д.
>> No.96362 Reply
>>96235
У него просто так принято. Он в интернете крутой.
>> No.96377 Reply
>>96193
Вектор - элемент линейно-векторного пространства. Линейно-векторного пространство - это два множества и операции, противопоставляющая каждой паре элементов первого множества элемент этого множества, каждой паре: элемент из второго и элемент из первого - элемент из первого, каждой паре из второго - элемент второго и ещё одна тоже каждой паре из второго - элемент из второго. Эти множества называют множеством векторов и множеством скаляров, а операции - сложением векторов, умножением вектора на скаляр, сложением скаляров и умножением скаляров, соответственно они и, чисто для удобства, обозначаются как + и *. Да такие, что для них верно, что:
Для любых скаляров верно, что:
1) (a+b)+c=a+(b+c)
2) Существует такой скаляр, обозначаемый 0, что
a+0=a
3) Для любого скаляра а существует скаляр, обозначаемый -а, такой, что
a + (-a) = 0
4) a+b=b+a
5) a*(b+c) = a*b + a*c   (приоритет - как обычно)
6) a*(b*c) = (a*b)*c
7) a*b = b*a
8) Существует такой скаляр, обозначаемый 1, что
a*1=a
9) Для любого скаляра а существует скаляр, обозначаемый 1/а, такой, что
а * (1/а) = 1

Для любых скаляров(маленькие буквы здесь - это скаляры для удобства, а большие - векторы) и векторов верно, что:
1) A+(B+C) = (A+B)+C
2) Существует вектор, обозначаемый 0(можно легко перепутать со скалярным нулём, как и можно перепутать операции, но это вовсе не значит, что они связаны сильнее, чем чётко описано, раз они одинаково обозначаются.), такой что
А+0 = А
3) Для любого вектора А существует вектор, обозначаемый -А, такой, что
А + (-А) = 0
4) А+B=B+A
5) a*(A+B)=a*A + a*B
6) (a+b)*A = a*A + b*A
7) a*(b*A) = (a*b)*A
8) 1*A = A
Последняя аксиома самая странная для непосвещённых, тем не менее. Для всех этих 17(складывал 8+9 полчаса) аксиом доказано, что ни одну из них нельзя вывести через другие.

Для многих аксиом есть куча названий, я их опустил, как бы раскрутив определения до конца.
>> No.96379 Reply
>>96377
А теперь видим, что тут вообще никаких якорей с реальностью нет. Математика применяется к реальности на свой страх и риск. Все объекты изучения математики - выдуманные, этакая игра в бисер. То, что яблоки можно считать числами - это просто опыт позволяет понимать. А в математике яблок нет. Есть только множества и операции со своими аксиомами.
>> No.96383 Reply
>>96379
А вот и нет. Математические понятия были получены путём абстрагирования от реальных объектов, и потому математика отражает свойства реальности.
>> No.96386 Reply
>>96379
Математика — часть физики. Физика — экспериментальная, естественная наука, часть естествознания. Математика — это та часть физики, в которой эксперименты дёшевы.
Тождество Якоби (вынуждающее высоты треугольника пересекаться в одной точке) — такой же экспериментальный факт, как то, что Земля кругла (т.е. гомеоморфна шару). Но обнаружить его можно с меньшими затратами.

http://www.ega-math.narod.ru/Arnold2.htm
>> No.96387 Reply
>>96383
Были получены, но не являются ими.
>>96386
> Математика — часть физики.
Нет, математика - самостоятельная абстрактная наука.
>> No.96388 Reply
>>96387
О, формалист-бурбакист в треде.

"Математика есть наука о доказательствах, доказательства это цепочки импликаций: (из А вытекает В, из В вытекает С) - цепочка; вывод: доказано, что из А вытекает С. Итак, самое главное - понять, что такое одна импликация. Вот ее определение. Пусть А и В - два произвольных высказывания. Если оба они верны, то говорят, что из А вытекает В".
На мой непросвещенный взгляд, такая точка зрения на импликации (а следовательно, и на доказательства, и на математику) - чистое мракобесие. При таком определении из того, что дважды два четыре, следует, что Земля вращается вокруг Солнца. Студента, понимающего выводы и доказательства подобным образом, уже бесполезно учить какой-либо естественной науке: мракобесие уничтожает естествознание как таковое. По этой мракобесной логике Галилея поделом наказывали: он ведь говорил о своих доказательствах вращения Земли и других подобных фактов совсем в другом смысле.
http://www.mccme.ru/edu/?ikey=viarn_burbaki


А в 10 лет я пытался добиться от отца объяснения (в школе нас учили без объяснений), почему умножение минуса на минус дает плюс.
Отец, как верный ученик Эмми Нётер, ответил: «Без этого нарушались бы аксиомы кольца вещественных чисел». Меня такой ответ не убедил: «А зачем нужно, чтобы выполнялись аксиомы?» Почему произведение минуса на минус дает плюс, я понял, когда сам решал такую задачу: «Сегодня прилив в городе N был в полдень. В котором часу он будет завтра?» Зная длину суток и месяца, легко вывести, что разница составит около 50 минут. А вот будет ли прилив на 50 минут раньше полудня или через 50 минут после него, – это выясняет именно « правило знаков».
http://www.mccme.ru/edu/viarn/2008/VM-1212.htm

Сами по себе математические построения пусты и безынтересны. Интерес представляет только та их часть, которая может быть использована в других науках.
>> No.96389 Reply
>>96388
> Сами по себе математические построения пусты и безынтересны. Интерес представляет только та их часть, которая может быть использована в других науках.
С этим я согласен. Но физика не единственная наука, которая интересуется математикой. Вообще формально описанные объекты одни и те же позволяют моделировать реальные объекты, которые сами по себе никак не связаны. Они именно моделируют - отбрасывают лишнее. Например, граф. В программировании вершины могут быть кусками кода, дуги обозначать, что после этого куска может начать выполняться этот. Могут вершины быть вхождениями переменных, а дуги будут характеризовывать информационные связи между ними(Анализ такого графа позволяет определить, можно ли программу распаралелить или поменять местами что-нибудь). В менеджмнете вершины могут быть делами, а дуга между делами - обозначать, что одно дело может быть начато только после выполнения предыдущего(см. Сетевое планирование). А в химии структура молекул изображается графами. А всё началось с мостов Кёнигсберга.

Я пошёл на математический факультет, так как меня убедили, что кодеру нужна математика. А вообще я хотел быть кодером. Конечно у меня есть коекакие теормех и УМФ, так что я могу помочь что-нибудь посчитать численно или запрограммировать физ-движок. Но я же не физик всё равно, так?
>> No.96390 Reply
>>96388
> Сами по себе математические построения пусты и безынтересны. Интерес представляет только та их часть, которая может быть использована в других науках.
Если ты привёл цитаты из ВИ в подтверждение этой мудрости, то у меня для тебя плохие новости.

>>96389
> что кодеру нужна математика
Ввели в заблуждение. Ну зачем кодерам алгебраическая К-теория или там контактная геометрия?
>> No.96391 Reply
>>96390
> Ввели в заблуждение. Ну зачем кодерам алгебраическая К-теория или там контактная геометрия?
Но мне же не сказали, что нужна вся целиком?
>> No.96392 Reply
>>96390
> Например, эти студенты никогда не видели параболоида, а вопрос о форме поверхности, заданной уравнением xy = z2, вызывает у математиков, обучающихся в ENS, ступор. Нарисовать на плоскости кривую, заданную параметрическими уравнениями (вроде x = t3 – 3t, y = t4 – 2t2) — задача совершенно невыполнимая для студентов (и, вероятно, даже для большинства французских профессоров математики).
Это кстати в твой огород камень, верно?
>> No.96393 Reply
>>96392
Эти два примера школьного уровня, ВИ тут гиперболизирует (если кто-то не понял).
Но, вообще, тратить значительную часть учебного времени (на первом курсе) на задачки в этом духе, если кто-то понимает о чём я, занятие почти преступное - с учётом того, сколько действительно важных вещей надо изучить в кратчайшие сроки.
>> No.96394 Reply
>>96390
Если ты не представляешь себе, чем занимаются программисты, то не выступай. Ознакомься с работами Александреску и вкури, как используются К-функторы в программировании.
>> No.96395 Reply
>>96377
> Вектор - элемент линейно-векторного пространства
"Я хочу положить такое определение".
> линейно-векторного
Лолдю
> 9) Для любого скаляра а существует скаляр, обозначаемый 1/а, такой, что
Пиздишь.
>>96386
Говно.
>>96387
> Нет, математика - самостоятельная абстрактная наука.
> наука.
Говно.
>>96390
> Ну зачем кодерам алгебраическая К-теория или там контактная геометрия?
С такой позиции много что в математике не нужно, вмести с занимающимися этим математиками.
Всякое бывает, кстати. http://www.mathnet.ru/php/seminars.phtml?presentid=313
>> No.96396 Reply
>>96394
Поподробнее, пожалуйста. И дай ссылочки на статьи, и правда ведь интересно.
>> No.96397 Reply
>>96377
Алсо, векторное пространство не только над полем бывает.
>> No.96398 Reply
>>96395
О, этого видео Манина я ещё не видел. Посмотрю как-нибудь, спасибо. А о чём там вкратце?
>> No.96399 Reply
>>96394
> вкури, как используются К-функторы в программировании
Что-то я ничего такого не нашла. Оно и понятно, нафига там К-теория?
Возможно, ты имел ввиду просто функторы, но это тривиальная вещь, которую можно и школьнику объяснить.
>> No.96440 Reply
>>96395
>Пиздишь
Я исправил это, а потом откатил исправления почему-то. Это пиздец. Заново, тогда.
Вектор - элемент линейно-векторного пространства. Линейно-векторного пространство - это два множества и операции, противопоставляющая каждой паре элементов первого множества элемент этого множества, каждой паре: элемент из второго и элемент из первого - элемент из первого, каждой паре из второго - элемент второго и ещё одна тоже каждой паре из второго - элемент из второго. Эти множества называют множеством векторов и множеством скаляров, а операции - сложением векторов, умножением вектора на скаляр, сложением скаляров и умножением скаляров, соответственно они и, чисто для удобства, обозначаются как + и *. Да такие, что для них верно, что:
Для любых скаляров верно, что:
1) (a+b)+c=a+(b+c)
2) Существует такой скаляр, обозначаемый 0, что
a+0=a
3) Для любого скаляра а существует скаляр, обозначаемый -а, такой, что
a + (-a) = 0
4) a+b=b+a
5) a*(b+c) = a*b + a*c   (приоритет - как обычно)
6) a*(b*c) = (a*b)*c
7) a*b = b*a
8) Существует такой скаляр, обозначаемый 1, что
a*1=a
9) Для любого скаляра а, не равного 0, существует скаляр, обозначаемый 1/а, такой, что
а * (1/а) = 1

Для любых скаляров(маленькие буквы здесь - это скаляры для удобства, а большие - векторы) и векторов верно, что:
1) A+(B+C) = (A+B)+C
2) Существует вектор, обозначаемый 0(можно легко перепутать со скалярным нулём, как и можно перепутать операции, но это вовсе не значит, что они связаны сильнее, чем чётко описано, раз они одинаково обозначаются.), такой что
А+0 = А
3) Для любого вектора А существует вектор, обозначаемый -А, такой, что
А + (-А) = 0
4) А+B=B+A
5) a*(A+B)=a*A + a*B
6) (a+b)*A = a*A + b*A
7) a*(b*A) = (a*b)*A
8) 1*A = A
Последняя аксиома самая странная для непосвещённых, тем не менее. Для всех этих 17(складывал 8+9 полчаса) аксиом доказано, что ни одну из них нельзя вывести через другие.

Для многих аксиом есть куча названий, я их опустил, как бы раскрутив определения до конца.
>> No.96451 Reply
Шолом, господа аноны.
Верно ли, что дополнение в R^n до не более чем счетного объединения его афинных подпространств коразмерности 2 линейно связно?
Я это, вроде, только что доказал, но не уверен.
>> No.96453 Reply
>>96440
А если векторное пространство рассматривается над телом?
>> No.96461 Reply
>>96440
Что за ебанистерия, блять. Первые 9 аксиом - аксиомы поля.
Существование нуль-вектора во второй части следует из аксиом поля и существования и дистрибутивности умножения вектора на сумму элементов поля, то есть, нихуя не аксиома, а сам нуль-вектор есть ноль поля умноженный на любой вектор.
Откуда откопал такое описание векторного пространства - закопай обратно поглубже, так пишут только мудаки.
>> No.96462 Reply
>>96453
а если векторное пространство над телом, то нет аксиомы 7) из первой части, только и всего.
А лучше нормальные книжки почитать, из >>96440 нихуя не понятно и некрасиво.
>> No.96463 Reply
В треде ещё остались математики, кстати?
Ответьте, пожалуйста, на >>96451
>> No.96465 Reply
>>96461
> Существование нуль-вектора во второй части следует из аксиом поля и существования и дистрибутивности умножения вектора на сумму элементов поля
Приведи соответствующие выкладки.
> Первые 9 аксиом - аксиомы поля.
Снизу написано, что определения специально раскручены.
>>96453
То ты сам перепишешь аксиомы. Это просто пример для того литературоведа, чтобы он понял, что есть математика.
> Алсо, векторное пространство не только над полем бывает.
Когда надо и очень хочется, всё что угодно бывает. Даже на английской вики определение даётся для поля. Может там дальше ещё уточнения были, но всё равно.
>> No.96467 Reply
>>96465
> Приведи соответствующие выкладки.
Пусть 0 - ноль базового поля, а - элемент поля, А - элемент векторного пространства над полем.
Тогда:
(а+0)А = аА + 0А для любого А,
Но а+0 = а по определению нуля, то есть:
(а+0)А = аА
и
аА = аА + 0А.
для любого А, то есть 0А - нуль-вектор.
Единственность нуль-вектора доказывается теми же рассуждениями, только в обратном порядке.
>> No.96472 Reply
>>96467
Я неудачно написал, там имелось в виду, что существует ноль такой, что для любого А, а не для любого А существет 0. Кванторы, конечно, в другом порядке.
>> No.96473 Reply
>>96467
> Единственность нуль-вектора доказывается теми же рассуждениями, только в обратном порядке.
Единственности нулей в аксиомах нет, это доказывается.
>> No.96476 Reply
File: Определение.JPG
Jpg, 102.38 KB, 1376×348 - Click the image to expand
edit Find source with google Find source with iqdb
Определение.JPG
>> No.96477 Reply
А можно короче.
Векторное пространство - унитарный модуль над полем.
>> No.96590 Reply
Есть простая задача:
> Ученик забыл последнюю цифру даты «Куликовской битвы» и поэтому называл её наудачу. Определите вероятность того, что до правильного ответа ему придётся отвечать не более 3 раз.
Правильный ответ: 3 / 10

Я решил её так:

Есть 3 гипотезы:

1. Ученик угадывает цифру с одного раза: P(H_1) = 1 / 10
2. Ученик угадывает цифру с двух раз: P(H_1) = (1 / 10) * (1 / 9) = 1 / 90
3. Ученик угадывает цифру с трёх раз: P(H_1) = (1 / 10) * (1 / 9) * (1 / 8) = 1 / 720

Вероятность наступления каждой из гипотез: 1 / 3

Тогда по формуле полной вероятности: (1 / 3) * ((1 / 10) + (1 / 90) + (1 / 720)) = 0.0375

Что я делаю не так?
>> No.96609 Reply
>>96590
> P(H_1) = (1 / 10) * (1 / 9)
   P(H_1) = (9 / 10) * (1 / 9)
> (1 / 10) * (1 / 9) * (1 / 8)
   (9 / 10) * (8 / 9) * (1 / 8)
Вероятность того, что он НЕ угадал, а не того, что угадал. Совсем неинтересно.
>> No.96611 Reply
>>96590
Вероятность угадать с первого раза 1/10
Вероятность угадать со второго раза 1/9
Вероятность угадать с третьего раза 1/8

То есть вероятность угадать с первых 3 попыток всего: 1/10 + 1/9 + 1/8 = 121/360. Примерно 1/3.

Что в этой задаче я понимаю не так?
мимогуманитарий
>> No.96616 Reply
>>96611
Расчёты отталкиватся от того, как мы считаем изначальные вероятности. Ты просто сразу счёл, что угадать с той или другой попытки - равновероятно. Мы сочли, что вероятность угадать n числел правильное - 1/n. То есть сочли равновероятными сами ответы ученика.
>> No.96617 Reply
>>96590
> Тогда по формуле полной вероятности: (1 / 3) * ((1 / 10) + (1 / 90) + (1 / 720)) = 0.0375
Я дальше не стал присматриваться, тут тоже прокол. Формула полной вероятности здесь не работает. Для этой формулы должно выполнятся, что гипотезы в объединении дают достоверное событие. То есть они должны исчерпать все варианты. Событие, вероятность которого ищется, - это объединением событий H, которые не пересекаются. Так что их надо было просто сложить.
>> No.96618 Reply
>>96617
> Тогда по формуле полной вероятности: (1 / 3) * ((1 / 10) + (1 / 90) + (1 / 720)) = 0.0375
Ну и на последок. Их вероятности не равны 1/3. Это ведь ты их считал. Или что? А умножаться они должны были тогда уж на вероятности угадывания при условии, что они уже верны. А они - это и сеть угадывания, то есть на единицы. Короче, это эпический бред.
>> No.96619 Reply
>>96611
> Вероятность угадать с первого раза 1/10
да
> Вероятность угадать со второго раза 1/9
Это условная вероятность, т.е. при условии, что первый раз не угадал. Нельзя её просто складывать с 1/10, нужно ещё умножить на вероятность не угадать в первый раз, т.е. 9/10 (формула Байеса).
> Вероятность угадать с третьего раза 1/8
Аналогично, нужно умножить на вероятности не угадать во второй и в первый раз

Получаем 1/10 + 9/10 x 1/9 + 9/10 x 8/9 x 1/8 = 3/10

Вообще решать задачу можно проще: считаем, что последняя цифра у даты тоже случайна (т.к. угадывание равновероятно) => вероятность того, что она попадёт в множество размера 3 равна 3/10
>> No.96620 Reply
>>96609
Спасибо, няша. Я всё понел.

(1 / 10) + (9 / 10) * (1 / 9) + (9 / 10) * (8 / 9) * (1 / 8) = 0.3
>> No.96622 Reply
>>96611
>>96619
В смысле, не формула Байеса, а просто определение условной вероятности
>> No.96625 Reply
>>96619
И от меня спасибо, теперь все понятно.
мимогуманитарий
>> No.96658 Reply
Анон, это я, что ли, тупой такой? Или Maple правда делали конченые наркоманы? Я вообще никогда не могу предсказать поведение ни редактора кода, ни интерпретатора. Я даже не знаю, что команда x:=y делает на самом деле и что вообще значит х тут и там!
>> No.96661 Reply
>>96658
Да, мы накоманы.
>> No.96667 Reply
>>96661
Под чем сидите?
>> No.96668 Reply
>>96667
Под линуксом.
>> No.96678 Reply
File: 476328946398.jpg
Jpg, 9.50 KB, 199×254 - Click the image to expand
edit Find source with google Find source with iqdb
476328946398.jpg
Анон, поясни за преобразования Лапласа.
Как из оригинала получить изображение и наоборот. Каковы их свойства. И коротко, где и как применяются.
Я честно проболел те две лекции, на которых их проходили.
>> No.96719 Reply
Между счетной и континуальной мощностями кто-то прячется!
>> No.96729 Reply
А где тот товарищ, который заявил об использовании К-функторов в программировании?

Неужели он это выдумал? Чтобы человек на имиджбордах рассуждал о вещах, в которых не разбирается? Нет, не могу в это поверить! Он просто решил затянуть интригу.
>> No.96732 Reply
Математики. Ко мне ночью пришла идея, которую я спешно записал и прочел сегодня утром. Это просто офигительно, и я заметил что пока никто в интернете ничего подобного не предлагал. Что мне с ней сделать? с идеей. Как реализовать её чтобы донести до других?
>> No.96740 Reply
>>96719
Кто-то толстый.
>>96729
Я тоже всё ещё заинтригован.
>> No.96741 Reply
Анон, подскажи пример функции 2 переменных, у которой смешанные частные производные второго порядка не равны друг другу (кроме примера Шварца из Википедии). Я так понимаю, достаточно потребовать разрывность одной из производных в какой-то точке и профит, однако самостоятельно составить такой пример не осилил... в теории я Лев Толстой, а на практике сами знаете
>> No.96790 Reply
>>96440
> А+B=B+A
Выводится без этой аксиомы.
>> No.96791 Reply
>>96790
Чтобы ни написали в треде, сегда кто-то скажет, что он не согласен, но никогда не опровергнет. Выведи и покажи.
>> No.96793 Reply
>>96790
Выведи.
>> No.96794 Reply
>>96790
Не выводится. >>96476 Не всякая группа - Абелева.
>> No.96797 Reply
File: 1365506418825.png
Png, 1.05 KB, 300×20 - Click the image to expand
edit Find source with google Find source with iqdb
1365506418825.png
>>96793
Вспомню - напишу.
>>96794>>96791
>> No.96802 Reply
>>96797
Для невырожденных матриц с их умножением верные первые три аксиомы, но нет комутативности. Ты откуда такой?
>> No.96824 Reply
Анон, вопрос, наверное, платиновый, и можешь меня отравить мышьяком, но я его задам.
Подскажи учебник по математике, рассчитанный на полного гуманитария.
С меня благодарность.
>> No.96825 Reply
>>96824
http://yadi.sk/d/dPI2g4Ct0xFMk
Гельфанд, Шень. "Алгебра".
>> No.96839 Reply
>>96824
Учебники с 6-го по 11-й класс раздобудь и начинай их читать. Раз уж ты гуманитарий, то книги ты читать научился. Проходишь их от корки до корки и понимаешь, что не такой уж ты гуманитарий, просто надо было внимательнее быть на уроках. Вот и всё.
>> No.96843 Reply
>>96802
Я анонимус.
>> No.96852 Reply
>>96741
Анон не сможет запилить пример?.. Бамп.
>> No.96910 Reply
File: Tyrande.Whisperwind.full.810156.jpg
Jpg, 837.00 KB, 2223×1361 - Click the image to expand
edit Find source with google Find source with iqdb
Tyrande.Whisperwind.full.810156.jpg
У меня есть ощущение того, что можно пренебречь постулированием коммутативости сложения векторов. Удовлетворительным аргументом было бы доказательство эквивалентности теории векторного пространства, где выражающая коммутативность формула не является аксиомой, теории векторного пространства, имеющей эту формулу в числе аксиом. Но поскольку пока в треде не предожено ни одной подходящей теории, я ограничусь косвенными аргументами.
sx означает, что x есть скаляр, и vx означает, что x есть вектор.
Если имеются обычные теоремы для равенства и аксиомы
(sx&sy&sz)→((x+y)+z=x+(y+z)),
sx→(x+0=x),
sx→(x+(-1*x)=0),
(sx&sy)→(x+y=y+x),
(sx&sy&sz)→((x*y)*z=x*(y*z)),
sx→(x*1=x),
(sx&~x=0)→∃w(x*w=1),
(sx&sy)→(x*y=y*x),
~0=1,
(vx&vy&vz)→((x+y)+z=x+(y+z)),
vx→(x+0=x),
vx→x+((-1)*x)=0,
sx→1*x=x,
(sx&sy&vz)→((x+y)*z=(x*z)+(y*z)),
(sx&vy&vz)→(x*(y+z)=(x*y)+(x*z)),
(sx&sy&vz)→((x*y)*z=x*(y*z)),
то выводимы
(vx&vy)→(x+y=(x+y)+0),
(vx&vy)→(0=(-1*(y+x))+(y+x)),
(vx&vy)→(0=((-1*y)+(-1*x))+(y+x)),
(vx&vy)→(x+y=(x+y)+((-1*y)+(-1*x))+(y+x)),
(vx&vy)→((x+y=((x+y)+((-1*y)+(-1*x)))+(y+x)),
(vx&vy)→((x+y=(((x+y)+(-1*y))+(-1*x))+(y+x)),
(vx&vy)→((x+y=((x+(y+(-1*y)))+(-1*x))+(y+x)),
(vx&vy)→((x+y=((x+0)+(-1*x))+(y+x)),
(vx&vy)→((x+y=(x+(-1*x))+(y+x)),
(vx&vy)→((x+y=0+(y+x)),
(vx&vy)→(x+y=y+x).
>> No.96914 Reply
File: 1111eee.jpg
Jpg, 29.67 KB, 1055×310 - Click the image to expand
edit Find source with google Find source with iqdb
1111eee.jpg
Доброанон, здравствуй!
Тут такое дело - у меня численные методы, делаю лабу, в Матлабе. Функицю ошибок я реализовал, а вот уже с производной методом интерполяции Лагранжа проблемы возникли. В лоб оно некорректно считается, препод сказал, конечно, мол, надо преобразовать!
Так вот, можете с этим помочь?
>> No.96926 Reply
>>96910
Чё ты плетёшь. Просто доказать надо аксиому из других, чтобы доказать её избыточность. Пипец, школьники увидели аксимы.
>> No.96927 Reply
>>96910
(vx&vy)→(x+y=(x+y)+0),
(vx&vy)→(0=(-1*(y+x))+(y+x))

сфигале? Сперва докажи, что обратный слева по сложению вектор является обратным справа.
>> No.96953 Reply
>>96926
>>96927
Я далёк от математики, поэтому если вы смеете обратиться ко мне, то прошу говорить как с обычным собеседником, воспринимающим
> Чё ты плетёшь
> Пипец
> сфигале
> Сперва докажи
как грубость.
Если >>96927 думает, что я написал доказательство, то напрасно. Я привёл только косвенные аргументы, надеясь, что используемая вами логика позволяет сделать соответствующие выводы.
>> No.96954 Reply
>>96927
> обратный слева по сложению
Я не хотел доказывать, я выводил.
>> No.96955 Reply
>>96954
Ты написал доказательный текст. Доказательный текст, по определению, последовательность соотношений такая, что каждое соотношение в нём является либо явной аксиомой, либо получено в результате применения схемы аксиом, либо выводится из предществующих соотношений по правилу вывода.
На каком основании ты написал (vx&vy)→(0=(-1*(y+x))+(y+x)) ?
>> No.96960 Reply
>>96910
Вот это
vx→x+(-x)=0
позволяет записать только
vx&vy→(x+y) + -(x+y)=0,
vx&vy→-(x+y) + --(x+y)=0,
vx&vy→--(x+y) + ---(x+y)=0
Отсюда не следует, что
vx&vy → -(x+y) + (x+y) = 0
и не следует, что
vx&vy → --(x+y) = (x+y)
>> No.96961 Reply
>>96960
Кроме того, я не увидел доказательства, что
vx&s(-1)→(-1)*x = -x
>>96953
> смеете обратиться ко мне
ЧСВ не нужно.
> как грубость.
Дык мы ребяты простые, пролетарии.

>>96954
Синонимы. Формула доказана, если существует вывод, в котором она стоит последней.
>> No.96964 Reply
>>96955
> Ты написал доказательный текст
Если такова рассматриваемая тобой формальная система. А я не ограничивался конкретной формальной системой, а указал только косвенный аргумент выводимости в подходящей системе.
> На каком основании ты написал (vx&vy)→(0=(-1*(y+x))+(y+x))
Полагая, что это выводимо в подходящей формальной системе.
>>96960
> позволяет записать только
> Отсюда не следует, что
Ты не упоминаешь конкретной логики, поэтому твои слова бессодержательны.
> >>96961
> Кроме того, я не увидел доказательства, что
> vx&s(-1)→(-1)*x = -x
Символы - и -x языку подходящей теории не требуются. Твои слова бессодержательны.
Возможно, ты имел ввиду, что вместе с аксиомой
vx→x+((-1)*x)=0
выводимо
vx→((-1)*x)+x=0.
Сообщаю, что последнее имеет простой вывод, опирающийся на (мета)теоремы для равенства.
> Синонимы.
Доказательство означает то же, что demonstration, а вывод — то же, что proof. Первое относится к неформальным рассждениям, а второе — к формальной области. Я косвенно аргументировал выводимость в подходящей формальной системе, а не наличие доказательства семантического свойства.
> обратный слева по сложению
означает свойство индивида и функции, то есть является семантическим.
>>96961
> ЧСВ не нужно.
Похоже, в твоём окружении принято этим попрекать. Но ведь здесь дело не в указании моей важной роли для тебя, а в том, что коли ты смеешь обратиться ко мне, то я советую придерживаться моих норм.
>> No.96966 Reply
>>96964
> Полагая, что это выводимо в подходящей формальной системе.
А почему ты решил, что это выводимо без коммутативности?
> Сообщаю, что последнее имеет простой вывод, опирающийся на (мета)теоремы для равенства.
Продемонстрируй.
>> No.96967 Reply
>>96964
ГОГ. Это ещё круче чем 0.(0)1
>> No.96968 Reply
>>96967
Няша пытается постичь формализм/логицизм. Зачем же над ним смеяться? Лучше помочь.
>> No.96976 Reply
File: P_adic_arithm.gif
Gif, 2.03 KB, 500×200 - Click the image to expand
edit Find source with google Find source with iqdb
P_adic_arithm.gif
>>96967
> 0.(0)1
Идея не так уж и бредова, какой может показаться. 0.(0)1 можно представить как эдакую подковку, два конца которой известны, а середина бесконечна. Что-то вроде p-адических чисел.
ruwiki://P-адические_числа
>> No.97020 Reply
Есть функция f(x):R->R и целое число х0. Ось иксов поделена на отрезки длины 1 целыми числами, "клеточки". Требуется найти, есть ли у f корни больше х0 и если есть, то какой клеточке принадлежит тот из них, который ближе всего к х0. То есть вычислить ближайший больший корень с точностью меньше 1. Есть какие-нибудь способы это находить для каких-нибудь f? Или можно ли определить, имеет ли функция корни на задонном отрезке или интервале?
>> No.97022 Reply
>>97020
Лол, гугл подсказывает:
> Запишите заданную функцию и приравняйте ее к нулю, например f(x) = 2х²+5х+2 = 0. Решите получившееся уравнение и найдите его действительные корни.
>> No.97030 Reply
>>97020
Такого способа для произвольных функций нет. Для полиномов можно использовать метод Штурма и метод Лобачевского-Греффе.
В остальных случаях как правило строят график на достаточно мелкой сетке, по графику определяют примерные координаты корней, после чего уточняют корни любым итерационным методом.
>> No.97031 Reply
Математики, в начале двадцатого века альфа-математиком был Пуанкаре, потом его сменил Гильберт. А кто лидер математического сообщества сейчас?
>> No.97034 Reply
File: BQps4u_gf8E.jpg
Jpg, 23.76 KB, 604×440 - Click the image to expand
edit Find source with google Find source with iqdb
BQps4u_gf8E.jpg
Умножение натуральных чисел не коммутативно. Вот контрпример на пике. Наверняка учитель с нашей кафедры.
>> No.97036 Reply
>>97034
С разморозкой. Иди-ка ты к Тифарету, у него три поста справедливого негодования.
>> No.97042 Reply
>>97031
Миша Вербицкий, очевидно же.
>> No.97051 Reply
File: 1010071753054.png
Png, 0.88 KB, 300×20 - Click the image to expand
edit Find source with google Find source with iqdb
1010071753054.png
>>96966
Похоже, достаточно (мета)теоремы для равенства (замена равного равным) и правила отделения.
vx→(-1*x)+x=(-1*x)+(x+0)
vx→(-1*x)+x=(-1*x)+(x+((-1*x)+(-1*(-1*x))))
vx→(-1*x)+x=(-1*x)+((x+(-1*x))+(-1*(-1*x)))
vx→(-1*x)+x=(-1*x)+(0+(-1*(-1*x)))
vx→(-1*x)+x=((-1*x)+0)+(-1*(-1*x))
vx→(-1*x)+x=(-1*x)+(-1*(-1*x))
vx→(-1*x)+x=0
>> No.97054 Reply
>>97051
Ок, сходу пробелов в выводе не вижу.
Аксиома "vx→x+((-1)*x)=0" непривычна, обычно вместо неё используется другая аксиома — о существовании и единственности противоположного элемента для x. То, что (-1)x = -x где -1 обратный к нейтральному по умножению скаляру скаляр, что -x единственен и что x и -x взаимообратны, обычно доказывается.
>> No.97060 Reply
File: 214458_html_faada68.gif
Gif, 101.00 KB, 1598×2048 - Click the image to expand
edit Find source with google Find source with iqdb
214458_html_faada68.gif
>>93583
Анон, суть вот в чем. Хочу понять суть дифференциирования и интегрирования через пределы последовательностей (без аналогий с изменением графика функции)
Что посоветуешь?
>> No.97061 Reply
>>97060
Википедию.
>> No.97062 Reply
File: edencelestesaban.jpg
Jpg, 55.82 KB, 445×576 - Click the image to expand
edit Find source with google Find source with iqdb
edencelestesaban.jpg
Пацаны, я в универе проболет 3 недели и теперь на контрольных обосрался. Очень нужна литература, где были бы доступно изложены темы:
-Ранг матрицы, ранг системы линейных уравнений
-Фундаментальная система линейных уравнений
-Интегралы. Здесь вообще пиздец. Ничего не понимаю, там какие-то страшные дроби с тригонометрическими функциями под знаком интеграла, радикалы, dx в числителе, внесение под дифференциал. А вообще не знаю, что это и как понимать, например, dx(sin x).
-Векторные пространства. Линейные операторы, матрицы линейных операторов. Проверка чего-то там на свойства колец (я на защите информации учусь).
Накидайте годноты по этим темам.
>> No.97066 Reply
>>97062
Стоит ли инженеру идти в аспирантуру и защищать кандидатскую или это имеет смысл только для работников гос НИИ?
>> No.97068 Reply
>>97062
Возьми любую книгу, то есть любую для рашкинской первокурсоты вообще. Алгебра - Курош, матан - лекции Письменного.

http://yadi.sk/d/FeBOATVX1ep6m
http://yadi.sk/d/Q8sMFpvJ0SMtb

http://yadi.sk/d/TVRB4HPb08kDQ
http://yadi.sk/d/TbAYh60q0SMIt

http://yadi.sk/d/9qMIaWMU0pBhV

Элементарщина, подходит под реквест:
http://yadi.sk/d/w9KUwR2I0SMYn
http://yadi.sk/d/UlT7nwzY0Vrck
>> No.97071 Reply
File: 1365861011547.png
Png, 1.15 KB, 300×20 - Click the image to expand
edit Find source with google Find source with iqdb
1365861011547.png
Надо внести исправление: нужно использовать не правило отделения, а правила, позволяющее для любых формул A, B получить из A B→A, и для любых формул A, B, C из A→B и B→С получить A→C.
Ну, этого достаточно, причём такое обычно является если не правилом вывода, то (мета)теоремой, так как выводы тавтологичны.
>> No.97072 Reply
>>97071
О чём ты говоришь?
>> No.97077 Reply
Положим, что f(x) стремится к бесконечности при x->бескон. Как доказать, что f(x)sinx тоже стремится к бесконечности? Это ведь верно? А если вместо sin будет g(x) периодическая, то можно ли тогда также говорить про f(x)g(x)?
>> No.97084 Reply
>>97077
Вольфрамальфа с тобой не согласен.
Если положить f(x) = x, то будет вот это: http://www.wolframalpha.com/input/?i=lim+x-%3E%2Binf+x*sin%28x%29
>> No.97085 Reply
>>97084
Ах да, я же там неверно себе представил стремление... Ок, тогда придётся всю задачу писать. Является ли относительно компактным множество функций { cos (sqrt(n)x) - cos (sqrt(n+1)x) | n=1,2,3,... }? По идее нужно юзать критерий Арцела. Мне кажется, что равностепенная непрерывность не должна выполняться. Как это доказать?
>> No.97086 Reply
привет, аноны.
подскажете с задачкой одной? а то что-то сообразить никак не могу, как решать.
Существует ли такое бесконечное множество A, которое можно представить в виде декартова произведения счетного семейства множеств Bi, каждое из которых имеет мощность меньше мощности A? Существует ли такое бесконечное множество A, которое нельзя представить в виде декартова произведения счетного семейства множеств Bi, каждой из которых имеет мощность меньше мощности A?
>> No.97106 Reply
>>97086
1)Континуум есть счетное декартово произведение конечных множеств (каждое действительное число записывается счетным набором цифр).
2)Счётное множество представляется аналогично, а бесконечные множества бывают только счётными и котинуальными, потому не существует, вроде так.
>> No.97107 Reply
>>97106
Лол.
Мощность множества всех подмножеств множества больше мощности самого множества; какова мощность множества всех подмножеств континуума?
>> No.97109 Reply
>>97107
Срать я хотел на ваш гиперконтинуум.
Когда я говорю "континуум", я имею ввиду любое множество, в которое вкладывается континуальное.
>> No.97110 Reply
>>97107
Хотя, да.
Гиперконтинуум, наверное, нельзя представить в виде счётного произведения континуумов.
>> No.97111 Reply
>>97109
> а бесконечные множества бывают только счётными и котинуальными
> Срать я хотел на ваш гиперконтинуум.
> Когда я говорю "континуум", я имею ввиду любое множество, в которое вкладывается континуальное.
Лол, одна история чудесатее другой.
Не бывает множества всех множеств, но существует бесконечно много множеств со всевозрастающей мощностью. Про кардинальные числа слышал что-нибудь? Алефы там, алеф-нуль, алеф-один, все дела.
ruwiki://Кардинальное_число
>> No.97113 Reply
>>97111
Я понял, да.
>> No.97138 Reply
File: _YTZlOUnuFs.jpg
Jpg, 36.16 KB, 580×326 - Click the image to expand
edit Find source with google Find source with iqdb
_YTZlOUnuFs.jpg
Пацаны а что такое гиперконтинуум?
>> No.97146 Reply
>>97138
подозреваю, что это расширение континуума
>> No.97192 Reply
>>97146
> подозреваю
Сначала тут были философы, теперь следователи.
>> No.97203 Reply
>>97192
Все мы - философы, но не все философы - мы.
>> No.97205 Reply
>>93583 Подскажите пожалуйста. Собираюсь поступать и мне надо знать, школьную программу математики. По специальности, будет очень много математики. По этому мне надо её очень хорошо знать. А я бросил её изучать в 4 классе. Вот и решил начать сначала. Доброанон дай совет как лучше это сделать и посмотри, правильно ли я нашел темы. Может, что упустил или полезно добавить еще чего. Есть ли какие секреты в решении. Мне как вообще, тупо по очереди брать школьные книги и всё подряд делать?

- Арифметика
- Изучение основ Алгебры
- Дробные числа, обыкновенные и десятичные дроби. Арифметические действия над дробями.
- Геометрические понятия, такие как площадь и объём простейших геометрических фигур.
- Понятия процента
- Полное изучение основ Алгебры.
- Делимость чисел, наиболее общий делитель и наименьшее общее кратное.
- Углуюленые знания в области обыкновенных дробей. Сложение вычитание с разными знаменателями.
- Умножение и деление обыкновенных дробей.
- Отрицательные числа. И основные арифметические действия на ними.
- Решение линейных уравнений с одной неизвестной
- Функции и многочлен.
- Свойства степеней с натуральным показателем.
- График линейной функции. Графики функций Х во второй и третей степени.
- Изучение формул сокращенного умножения. И использование их для преобразования и разложения на множители различных выражений.
- Решение системы линейных уравнений как аналитическими, так и графическими способами.
- Изучение рациональных дробей, их свойства и действия которые можно производить с ними.
- Знакомство с понятием арифметического квадратного корня и его свойствами.
- Решение квадратных уравнений с использованием формулы корней квадратного уравнения и применение полученых знаний для решения задач.
- Изучение свойства числовых неравенств и действия над ними.
- Изучение степени с целем показателем и её свойства.
- Изучение методов решения уравнений и неравенств. Использование аналитические и графические способы решения.
- Рассмотрение уравнений и неравенств как с одной так и с двумя переменными.
- Подробнейшим образом рассматривается квадратичная функция и её свойства. Именно при изучении этого раздела рассматриваются различные способы решения уравнений второй степени.
- Основные сведенья об арифметической и геометрических прогрессиях.
- Изучение элементов комбинаторики и теории вероятности.
- Изучение основ математического анализа. Перво-наперво изучается одно из основных понятий математического анализа - числовая функция.
- Изучение свойств различных числовых функций и приведение примерного плана для исследования функций.
- Изучение основные тригонометрические функции их свойства, тригонометрические уравнения и способы их решения.
- Установление закономерности между различными тригонометрическими функциями, и их использование для преобразования тригонометрических выражений.
- Понятие производной и её применение при исследовании функций. А также выясняется её геометрический и механический смысл.
- Углубленое изучение понятий степени и корня.
- Рассмотрение степенной, показательной и логарифмической функции. Исследование их графиков и выяснение основых свойств.
- При изучении лагарифмической функции вводится понятие логарифма и его свойства.
- Рассмотрение основных логарифмических неравенств и уравнений и методы их решения. Помимо этого учаться решать системы уравнений и неравенств.
- Курс анализа оканчивается изучением первообразной и определение интеграла.
- Изучение таблиц простейших интегралов и применение их для вычесления площадей плоских фигур.
>> No.97214 Reply
Расскажи про НМУ, Матемач.
Как я понял (из статьи на Лурочке) — ты выбираешь какие-то курсы, разделы математики. Плюс есть обязательные.
Чтобы сдать выбранные предметы — надо лишь решить выданный листочек с заданиями и всё? А потом, в конце обучения, сделать дипломный проект? И занятия возможно совсем не посещать?
>> No.97226 Reply
File: Cdg_60.jpg
Jpg, 46.58 KB, 852×480 - Click the image to expand
edit Find source with google Find source with iqdb
Cdg_60.jpg
>>97205
Тебе сюда, няша:
http://dobrochan.org/rf/res/415483.xhtml#i418190
Олсо. Матемач, покритикуй список. Я хочу зделать идеальный для тех, кто только хочет начать изучать математику и постить его каждый раз как увижу такой запрос.
Чёрт, тут же был ещё один нормальный пост сверху. Куда, куда он делся?
>> No.97228 Reply
>>97214
Рассказываю.
Чтоб сдать выбранные предметы, нужно сдать зачёт и экзамен.
В зачёт входит, обычно, сдача задач из листков (бывает, что нет).
Экзамен обычно подразумевает письменное решение задач на экзамене (бывает, что нет).
Занятия можно совсем не посещать, но лучше посещать, по крайней мере, листки сдавать в течение, а не в конце на зачёте, иначе есть риск потерять нить, ибо программы интенсивнее мехматовских раза в два или больше (и лучше при этом во столько же).
Все курсы открыты для всех желающих (кроме ин. языков), никакой записи не нужно (но можно).
Словосочетание "дипломный проект" звучит по-говноедски, отвыкайте от такого.
Коротко: НМУ - жидовская секта, созданная с целью подрыва столпов арийской математики.
>> No.97232 Reply
Матемач, спали годные книжки по матану, аналиту и матлогике, фихтенгольца и бека не предлагать.
>> No.97235 Reply
>>97232
Читай классиков же. >>rf/419369
>> No.97237 Reply
>>97214
(дополнение к >>97228)
> есть обязательные
они условно "обязательные", ходить нужно куда хочешь и можешь. Например курс "геометрии" в первом семестре почти всегда дурацкий и на него можно (и лучше) не ходить. А алгебру и анализ придётся выучить в любом случае, они обязательные по самой своей сути.

>>97226
чуть позже посмотрю подробнее, что ты скомпилировал %% :3 %%
>> No.97238 Reply
>>97237
ололо я проебал разметку
>> No.97240 Reply
File: 19392611974c31ef1ab20049.26433488.jpg
Jpg, 24.79 KB, 450×320 - Click the image to expand
edit Find source with google Find source with iqdb
19392611974c31ef1ab20049.26433488.jpg
>>97238
Добра тебе, няша. Почему-то захотелось тебя обнять.
Держи зайца.
>> No.97242 Reply
File: bashful924.jpg
Jpg, 41.00 KB, 340×264 - Click the image to expand
edit Find source with google Find source with iqdb
bashful924.jpg
>> No.97280 Reply
File: 1366037164006.png
Png, 0.76 KB, 300×20 - Click the image to expand
edit Find source with google Find source with iqdb
1366037164006.png
>>97226
Для математики не хватает только Зорича и Ильина и Позняка.
А шишек никак не миновать
>> No.97281 Reply
>>97280
> Ильина и Позняка
Но зачем читать этот неоригинальный учебник? Лучше Рудина или даже Фихтенгольца (хоть обе эти книжки и устарели одна на пятьдесят, другая на сто пятьдесят лет).
>> No.97287 Reply
>>97226 Большое спасибо бро. Добра.
>> No.97293 Reply
нужно срочно разобраться с LaTexом. Поставил ProTex, но там были траблы с русским. Babel какой-то костыльный. Услышал про XeTex и поставил Tex live 2012. Теперь не понимаю как заставить xetex работать из студии. Да и вообще слабо понятно как это должно работать. Все это я затеял для быстрого ведения конспекта на лекции.
>> No.97298 Reply
>>97293
Попробуй LyX. Для лекций думаю будет отличным решением (WYSIWYM + можно вводить код TeX).
>> No.97309 Reply
>>97293
На формулах не будешь успевать
>> No.97316 Reply
Привет, анон. Относительно какой операции не замкнуто множество комплексных чисел?
>> No.97317 Reply
>>97309
Да я в ворде кое как успевал, но с трудом. Да и ощущалось это как лишний геморой.
>>97298
Попробовал, спасибо. Надо бы получше разобраться, но пока что получается не очень быстро. Да и графики не порисуешь особо.
>> No.97354 Reply
File: 1366100399783.jpg
Jpg, 77.49 KB, 937×960 - Click the image to expand
edit Find source with google Find source with iqdb
1366100399783.jpg
Утащил с форчана, а ответа не знаю.
>> No.97360 Reply
>>97354
Вариант B- 50%
4 варианта, из которых два верные. Решение то случайным порядком выбирается исключительно в голове, а ответ даем сознательно
>> No.97361 Reply
>>97360
> 4 варианта, из которых два верные
Но ты же только что сказал, что верным является только один вариант B.
>> No.97362 Reply
>>97316
Рассмотрим множество всех овощей, будем его обозночать Ω. Рассмотрим операцию ω:СхС->Ω по такому правилу: если z=/=w и z=/=42, то ω(z,w)=огурец, если z=w и z=/=42 - помидор, а при z=42 - перчик.
>> No.97363 Reply
>>97362
Да, заметим, что ни перчик, ни огурец, ни помидор не принадлежат множеству комплексных чисел, так как они не являются комплексными числами.
>> No.97364 Reply
>>97362
Ок. Что-нибудь более интересное знаешь?
>> No.97365 Reply
>>97361
Но если верен только B, то шанс правильно ответить - 25%, и тогда правильные ответы - A и D. Но тогда шанс правильного ответа уже 50%, и верным ответом становится снова B. Так, все, я запутался.
>> No.97366 Reply
File: Bertrand_Arthur_William_Russell.jpg
Jpg, 26.82 KB, 233×404 - Click the image to expand
edit Find source with google Find source with iqdb
Bertrand_Arthur_William_Russell.jpg
>>97365
ке-ке-ке
>> No.97372 Reply
>>97354
> Утащил с форчана, а ответа не знаю.
Не имеет смысла говорить о вероятностях, пока не введено какое-либо вероятностное распределение.
Следующий.
>> No.97373 Reply
>>97372
Самый умный что ли? Очевидно же, распределение тут uniform.
>> No.97374 Reply
>>97373
Распределение - это функция. Описывая функцию, падаван, область определения указывать обязан ты. Ибо кортежем трёхместным функция является. Таки я ещё умнее.
>> No.97379 Reply
>>97281
Потому что математика. Не как что-то хорошее. Просто я тяготею к основаниям и не люблю неформальную математику. Вот и называю махровые примеры.
>> No.97396 Reply
>>97374
Ты издеваешься? Дискретная функция f, определена для аргументов A, B, C, D. f(A)=f(B)=f(C)=f(D)=0.25
>> No.97397 Reply
File: интеграл.png
Png, 2.28 KB, 656×224 - Click the image to expand
edit Find source with google Find source with iqdb
интеграл.png
Анон, вот сейчас вычисляю интегралы, но вот с одним примером не знаю что делать.Чему равен ln(x^0.5)
>> No.97399 Reply
>>97397
ln(x)/2
>> No.97404 Reply
>>97396
> Дискретная функция f, определена для аргументов A, B, C, D. f(A)=f(B)=f(C)=f(D)=0.25
В условии этого нет.
>> No.97406 Reply
>>97404
Это подразумевается. Когда тебе про монетку или игральную кость говорят, ты тоже будешь распределение вероятностей на орла или решку/1-6 требовать?
>> No.97407 Reply
>>97374
только распределение это не функция. Это элемент дв-ного пространства.
>> No.97408 Reply
Здравствуйте. Был бы рад, если бы мне оказали помощь в одном задании. Сам я над ним бьюсь уже несколько дней и ничего не могу добиться. Надо найти все натуральные числа, сума чисел которых будет в 11 раз меньше чем само число. Я смог найти только число 198, но мне утверждают что их больше. Буду рад помощи.
>> No.97409 Reply
>>97406
Нет, это как раз не подразумевается, это и вызывает парадокс.
>> No.97416 Reply
>>97409
Если это подразумевается, то тоже парадокс.
>> No.97428 Reply
File: 2292a057a2e5d5cdc2b9d8679e126aa4.jpg
Jpg, 533.82 KB, 794×1076 - Click the image to expand
edit Find source with google Find source with iqdb
2292a057a2e5d5cdc2b9d8679e126aa4.jpg
>>97408
> Буду рад помощи.
Уравнение составил?
Обозначим искомое число как b.
b = a(0)*10^0 + a(1)*10^1 + ... + a(n)*10^n, где n — число разрядов в числе, а a — целое положительное одноразрядное число.
Нужно найти все числа, для которых выполняются условия:
*b* = 11*(a(0) + a(1) + ... + a(n));
*b* = a(0)*10^0 + a(1)*10^1 + ... + a(n)*10^n;
(10^(n)-1) < b =< (10^(n+1)-1);
a(n) != 0;
истинно.
И конечно все числа натуральные.
Ну ты понел. Дальше делаешь так:
[a(0)*10^0 + a(1)*10^1 + ... + a(n)*10^n] - [11*(a(0) + a(1) + ... + a(n))] = 0;
Одно- и двухзначные числа выпадают сразу, ведь:
-10*a(0) - a(1) < 0;
Для трёхзначных чисел условие будет:
10*a(0) + a(1) = 89*a(2);
Для четырёхзначных уже:
10*a(0) + a(1) = 89*a(2) + 989*a(3);
Что невозможно, так как при максимальной оценке для четырёхзначных чисел:
10*9 + 9 != 89*0 + 989*1;
Равенство уже не выполняется. И конечно — для более многозначных.
Соответственно задание сводится к вот этому уравнению:
10*a(0) + a(1) = 89*a(2);
Можешь уже проверять с a(2), подставляя числа от 1 до 9. Но 10*a(0) + a(1) в любом случае меньше или равно 99 из-за ограничений на значения a.
Ответ, как уже найдено — число 198.
Вывод: тебя обманывают. Унижай, гони, оскорбляй их.
>> No.97440 Reply
>>97428
Спасибо большое, тоже составил уравнение и на основе ограничений нашел число, но у меня постоянно было чувство, что я что-то упускаю. Но мне продолжают упорно утверждать, что там больше одного ответа. И еще вопросик, как научиться так логично и последовательно излагать свои мысли в математике?
Даже не знаю как благодарить.
>> No.97526 Reply
File: ae66.png
Png, 540.16 KB, 707×1000 - Click the image to expand
edit Find source with google Find source with iqdb
ae66.png
>>97440
> Но мне продолжают упорно утверждать, что там больше одного ответа.
О, это странно.
Но пути ветра уравнения не врут же. Если они составлены без ошибок.
Каким образом «упорно утверждают, что там больше одного ответа»? Называют ещё ответы? Кто и где? Преподаватель? Задачник —> скриншот сюда?
> логично и последовательно
Читай & практикуйся же! Со временем само приходит.
А так тебе в /rf/:
http://dobrochan.org/rf/res/415483.xhtml#i419369
>> No.97531 Reply
>>97526
Эту задачку дала нам преподаватель на курсах. У нас будет тест для тех, кто на курсах год, а мы только 4 месяца, по этому она решила дать нам несколько заданий, что будут на тесте. Это одно из них. Все кто его решал в нашей группе утверждают, что там один ответ. Она же говорит что эту задачу дают на тесте( вопрос с открытым ответом) уже не первый год и ее решили только два раза, но как, она отказывается говорить.
>> No.97535 Reply
>>97526
>>97531
Нуль забыли, имхо.
>> No.97536 Reply
File: Безымянный.png
Png, 3.37 KB, 345×113 - Click the image to expand
edit Find source with google Find source with iqdb
Безымянный.png
Пытаюсь найти главную часть некоторой функции, в которой находится вот такой логорифм.
Необходимо преобразовать функцию таким образом, чтобы все иксы объединились в один икс некоторой степени.
Но как это сделать, до меня не доходит.
Может, поможете?
>> No.97537 Reply
>>97536
Или всё проще, и значение под логорифмом слопывается в единицу?
>> No.97538 Reply
>>97536
Разложи логарифм в ряд Тейлора.
>> No.97551 Reply
>>97535
Какой нуль? Где?
>> No.97553 Reply
>>97551
> Надо найти все натуральные числа, сума чисел которых будет в 11 раз меньше чем само число.
0 = 0*11. Если натуральный ряд начинать с нуля, конечно.
>> No.97569 Reply
>>97553
Ну и нестрогий порядок.
Решение не читал, но верю анону. Так что 0 являеься объяснением.
>> No.97570 Reply
>>97569
> являеься
является
>> No.97577 Reply
File: Untitled-1.jpg
Jpg, 8.83 KB, 445×318 - Click the image to expand
edit Find source with google Find source with iqdb
Untitled-1.jpg
Имеет ли смысл интеграл с такими пределами?
>> No.97578 Reply
>>97577
Очевидно, нет. Это интеграл по прямой, а не по плоскости.
>> No.97581 Reply
>>97577
1/2(1^2 - i^2).
>> No.97582 Reply
File: Untitled-1.jpg
Jpg, 11.93 KB, 684×476 - Click the image to expand
edit Find source with google Find source with iqdb
Untitled-1.jpg
>>97578
А как-нибудь вот так его рассматривать нельзя?
>> No.97583 Reply
>>97582
Но зачем?
>> No.97584 Reply
>>97582
Рассматривают просто интегралы по всяким кривым, областям, поверхностям и т.д., подписывая под крючком знак, обозначающий множество, по которому интегрируется.
>> No.97586 Reply
> натуральный ряд начинать с нуля
Лопни мои тестикулы, а ведь это таки ответ!
Как я понимаю, российское образование нонче переходит на современную математику.
Некоммутативную алгарифметику: http://dobrochan.org/u/res/93583.xhtml#i97034
И теперь считает ноль элементом множества натуральных чисел, как и надо.
Хорошо, чо. Скоро начнут преподавать общую топологию в восьмом классе и эргодическую теорию в одиннадцатом, начиная сразу с интегрального исчисления.
На самом деле мне грустно, ведь в школе мне девять лет повторяли, де ноль это никак не натуральное число, давая определение «этим можно считать яблоуки» вместо определения по мощностям множеств или Пеано.
И я на автомате отбросил нуль, даже не рассматривая его как натуральное число, хотя знаю про два написанных выше определения.
Пойду отсюда гнить в /rf/, поджав горящие кальсоны.
//Дополняем, переделываем:
[a(0)*10^0 + a(1)*10^1 + ... + a(n)*10^n] - [11*(a(0) + a(1) + ... + a(n))] = 0
Пусть n = 1, тогда:
10*a(0) = 0;
Если a(0) = 0, тогда равенство истинно и '0' является искомым числом.
Пусть n = 2, тогда:
10*a(0) = -a(1);
Равенство ложно при любых а.
Пусть n = 3, тогда:
10*a(0) + a(1) = 89*a(2);
Равенство истинно при a(2) = 1, a(1) = 9 и a(0) = 8;
Пусть n = 4, тогда:
10*a(0) + a(1) = 89*a(2) + 989*a(3);
Но левая часть уравнения ограничена условием 10*a(0) + a(1) =< 99 при a(0) & a(1) (= [0; 9] U N,
соответственно равенство ложно при любых a.
Дальше, с помощью метода математической индукции, доказываем что при любых значениях n больше трёх равенство будет ложным. Этим мы удивим преподавателя и будем купаться в ништяках репутации умняши, когда будем учиться в университете.
>> No.97598 Reply
File: 1307696095718.jpg
Jpg, 201.57 KB, 800×600 - Click the image to expand
edit Find source with google Find source with iqdb
1307696095718.jpg
> Следующий, кто спросит, почему 0.(9) = 1
не читай - отвечай
Близится колобок по диффурам, а с ним и куча вопросови анальной боли. Можно ли рассчитывать на сапортов в этом треде, учитывая что в шапке эта фраза?
>> No.97604 Reply
>>97598
Ты написал так, что ничё не поймёшь, потому, что любишь, когда тебя переспрашивают?
>> No.97609 Reply
>>97604
Когда переспрашивают - не люблю.
Написал так, потому что либо я еблан, либо одно из двух.
Короче говоря: узрев сию фразу, я усомнился в уровне возможной математической помощи местных обитателей.
>> No.97610 Reply
>>97604
Это вопрос?
>> No.97617 Reply
>>97609
1=0,(9) - это местная "теорема Ферма", которую всё время кто-нибудь пытается опровергнуть, она упомянута из-за бывшего большого количества флуда сомневающися. Можно задать вопрос, но на него могут ответить, а могут и не ответить. Я думаю, тут у всех уровень курса 2-3 усреднённо.
>> No.97619 Reply
>>97617
При этом ни один из опровергальщиков не приводил определения числа.
>> No.97621 Reply
Ноль - это не натуральное число, кстати
>> No.97622 Reply
>> No.97623 Reply
>>97622
так кто же читает русскую вики по математике!!
>> No.97624 Reply
>> No.97625 Reply
>>97624
Окей, но "There is no universal agreement about whether to include zero in the set of natural numbers"
>> No.97626 Reply
>> No.97627 Reply
>>97626
это на каком?
>> No.97628 Reply
File: joker-e1276968083519.jpg
Jpg, 61.77 KB, 550×550 - Click the image to expand
edit Find source with google Find source with iqdb
joker-e1276968083519.jpg
>>97625
> Ноль - это не натуральное число, кстати
>> No.97661 Reply
В чём важность понимания нуля как натурального числа? Почему не начать натуральные с -1 или 2?
>> No.97679 Reply
File: фрегерассел.PNG
Png, 5.28 KB, 393×120 - Click the image to expand
edit Find source with google Find source with iqdb
фрегерассел.PNG
>>97661
Натуральные числа можно определить как ординалы.
>> No.97710 Reply
>>97661
Если с минус единицы, то сложение даст противоположное к каждому из чисел. Если с двух, то будут другие своства делимости.
>> No.97711 Reply
>>97710
> Если с минус единицы, то сложение даст противоположное к каждому из чисел
Нет, нифига, например (-1)+(-1) не будет определено.

>>97661
С нулём натуральные числа превращаются в моноид по сложению. Мелочь, а приятно.
>> No.97719 Reply
>>97679
Разве это удобно?
Как после этого определять арифметические операции и доказывать их свойства?
>> No.97728 Reply
>>97719
Элементарно же.
a + 1 = a∪{a}

ruwiki://Ординал
>> No.97732 Reply
>>97728
Нет, как умножить a и b, например?
Почему бы не сделать так:
1 - это кружочек: o.
2 - это 1 и кружочек: 2 = 1o = oo
3 - это 2 и кружочек: 3 = 2o = ooo
И т.д.

Чтобы сложить два числа, просто записываем их рядом: 3 + 2 = ooooo
Чтобы умножить два числа, заменяем каждый кружочек в 1-м на запись второго: 3 x 2 = (oo)(oo)(oo) = oooooo

Ноль определяем вместе с отрицательными числами:
0 = множество классов эквивалентности пар (a, b), т.ч. a = b
-1 = множество классов эквивалентности пар (a, b), т.ч. a + 1 = b
и т.д.
>> No.97733 Reply
>>97732
> как умножить a и b, например?
Традиционно через +1.
> Почему бы не сделать так
У тебя равенство, классы эквивалентности, пары и т.п. не формализованы, так что тебе всё равно придётся теорию множеств вводить.
>> No.97737 Reply
>>97732
Почему 0.(9) = 1?
>> No.97740 Reply
>>97737
41.(9)
>> No.97741 Reply
>>97740
Доказать можешь, нет?
>> No.97742 Reply
File: langoleryi.jpg
Jpg, 19.29 KB, 600×452 - Click the image to expand
edit Find source with google Find source with iqdb
langoleryi.jpg
>> No.97743 Reply
>>97742
Доказывай.
>> No.97744 Reply
>>97743
Лол. Мы это в 5 классе доказывали.
>> No.97745 Reply
>>97744
Ок, повтори доказательство. Со всеми определениями, не пропуская логических ходов.
>> No.97746 Reply
>>97743
0.(9) = 0.(1)9 + 0.(1) - 0.(1) = 0.(1)(9+1) - 0.(1) = 0.(1)*10 - 0.(1) = 1.(1) - 0.(1) = 1
>> No.97747 Reply
>>97745
Почему бы тебе его в книге не поискать?
>> No.97748 Reply
>>97746
Э, нет. Ты сперва определения бесконечной дроби дай, определение сложения и определение умножения. Пользоваться неопределёнными вещами любой человек с улицы может.

>>97747
Ссылку на книгу и номер страницы в студию.
>> No.97749 Reply
>>97746
ЛОЛО
Вы тут все такие пятиклассники?
>> No.97750 Reply
>>97749
Сделай лучше, покажи класс.
>> No.97751 Reply
File: 80939726_Smesharik_Nyusha_Liza.jpg
Jpg, 35.14 KB, 400×400 - Click the image to expand
edit Find source with google Find source with iqdb
80939726_Smesharik_Nyusha_Liza.jpg
Ну где же вы, джынтыльмены?
>> No.97752 Reply
>>97751
Никто не будет тут пересказывать всю математику, она большая. Ищи в книгах.
>> No.97753 Reply
>>97752
А всю и не надо, просто обоснуй, почему 0.(9) = 1.
>> No.97754 Reply
>>97752
> Ищи в книгах
> Ссылку на книгу и номер страницы в студию.
>> No.97755 Reply
Сторонники того, что 0.(9) таки равно единицу пока сливают!
Ну а мы внимательно следим за развитием событий %%на самом деле нет%%, что же будет дальше, интриги сенсации
>> No.97756 Reply
>>97755
да что ж такое! опять разметку проебал
>> No.97757 Reply
>>97753
>>97755
Семён такой семён
>> No.97758 Reply
>>97755
Если ты такой крутой, то сам обоснуй это утверждение.
>> No.97759 Reply
>>97758
Нашел доказательство, понятное даже школьнику:
1/3=0.(3)
0.(9)=0.(3)*3
(1/3)*3=1
0.(9)=1
>> No.97760 Reply
>>97759
> 0.(9)=0.(3)*3
Сфигали?
>> No.97761 Reply
>>97759
Ты не определил бесконечную десятичную дробь, равенство бесконечных десятичных дробей, равенство бесконечной дроби и правильной дроби, умножение бесконечных дробей на целое число и некоторые другие вещи.
> 1/3=0.(3)
А вот это с чего ты взял? Это ведь всё равно что сказать 0.(9) равно 1 по определению, это не доказательство.
>> No.97762 Reply
>>97761
Ну, давай поиграем. Я надеюсь, что ты пришел поиграть или потроллить, а не на полном серьезе поднимаешь эту муть.
Пусть 0.(9)!=1. Тогда 1-0.(9)=r!=0. Тогда в записи числа r хотя бы на каком-то i-том месте будет не 0. Но тогда в числе 0.(9)+r на i-м месте стоит цифра >=0, а на i+1-м месте стоит цифра >=1, что невозможно, так как 0.(9)+r=1.
>> No.97771 Reply
>>97762
Ты ведь так и не определил бесконечные дроби, поэтому непонятно, как ты их сравниваешь и как совершаешь с ними действия. А ещё ты сравниваешь 0.(9) с 1, хотя 1, вроде как, не является бесконечной дробью.
> Тогда в записи числа r хотя бы на каком-то i-том месте будет не 0.
Почему?
> Но тогда в числе 0.(9)+r на i-м месте стоит цифра >=0, а на i+1-м месте стоит цифра >=1
Откуда это следует? Как то, что
> 0.(9)+r=1
противоречит вышезаявленному?
>> No.97777 Reply
>>97771
Раз ты задаешь вопрос про бесконечные дроби, то, стало быть, ты уже считаешь их определенными, иначе твой вопрос не имеет смысла.
>> No.97788 Reply
Бля, опять(
>> No.97790 Reply
Пфффф... Будем называть действительным числом целое число a0 и отображение a из N(\{0}) в {0,1,2,3,4,5,6,7,8,9}. (Всюду далее епсилон - рациональное.) Будем говорить, что действительное x равно рациональному q тогда и только тогда, когда для любого епсилон>0 найдётся i0 натуральное(не 0) такое, что для любого i>i0 будет выполнятся |S[i]-q|<епсилон. Под S[i] понимаем сумму по j от 1 до i слагаемых a[j]/(10^j). Будем говорить, что x<q(x>q) тогда и только тогда, когда существует епсилон>0, такое, что для любых i0 натуральных найдётся i>i0 такое, что S[i]+епсилон<q(S[i]-епсилон>q). Тогда будем называть равными такие действительные числа x и y, что не существует числа q рационального такого, что x<q<y или такого, что y<q<x.

И докажем, что если два действительных числа x и y равны одному рациональному q, то они равны:

Предположим, что это не так и существует р такое, что x<p<y(возьмём такой порядок для определённости) Тогда
Найдётся епс1 такое, что для любого i0 найдётся i>i0 такое, что Sx[i]+епс1<p
Найдётся епс2 такое, что для любого i0 найдётся i>i0 такое, что Sy[i]-епс2>p
Пусть епс=max{епс1,епс2} Тогда
Sx[i]+епс<=Sx[i]+епс1<p
Sy[i]-епс>=Sy[i]-епс2>p
Найдётся i0(максимум между двумя для x и y) такое, что для любых i>i0
|Sx[i]-q|<епс
|Sy[i]-q|<епс
Тогда Sx[i]-p < -епс и Sy[i]-p>епс, а q-Sx[i]<епс и q-Sy>-епс
q-p = q-Sx[i]+Sx[i]-p < епс - есп = 0
q-p = q-Sy[i]+Sy[i]-p > -епс + епс = 0
0<q-p<0 - противоречие. Таким образом такого p нет и x=y.

То, что рациональная 1 равна действительной 1 очевидно, ведь все S[i]=1 и |S[i]-1| всегда =0. Докажем, что 0,(9) равна рациональной 1 и она тогда окажется равной действительной:

Докажем сначала, что 1 рациональная равна для любого i [сумма по j от 1 до i слагаемых 9/(10^j)] + 1/(10^i). По индукции:
База: 1=10/10=9/10 + 1/10
Индуктивный переход: 1 = сумма(1..i,9/(10^j))+1/(10^i)=сумма(1..i,9/(10^j))+10/(10^(i+1))=
сумма(1..i,9/(10^j)) + 9/(10^(i+1)) + 1/(10^(i+1))=сумма(1..i+1,9/(10^j)) + 1/(10^(i+1))

Теперь |сумма(1..i,9/(10^j))-1|=1/(10^i)<епс
i*ln(1/10)<ln(епс)
i0=ln(епс)/ln(1/10)
Для любого епс>0 для любого i>i0 выполняется |сумма(1..i,9/(10^j))-1|<епс
Так что 0,(9)=1
>> No.97792 Reply
>>97790
Епта, сборище идиотов.
>> No.97794 Reply
>>97792
Ну давай свои единственно правильные определения.
>> No.97802 Reply
Я вот думаю. Что если доказательство - это только для слабых. Для сильных - навязывание. В самом деле не от того ли задроты дезадаптированны в обществе, что они всё учатся доказывать тогда, как успехные люди учатся убеждать используя когнитивные искажения, а не логику.
>> No.97811 Reply
сап, матемач
посоветуй книжулину по финансовой математике и о чем-нибудь околоэкономическом-математическом, а то читать нечего.
>> No.97814 Reply
>>97811
<ирония>Считать деньги - низкое дело. Займись лучше изучением новых миров топологии и теории супер-струн! А пошлую реальность оставбь быдлу.</ирония>
>> No.97834 Reply
Анон,
пытаюсь вкурить в применения интеграла. Подскажи, книжку, где бы выводились площади и объёмы всяких ебанутых кривых и фигур. Сейчас меня конкретно интересует, как из записи в декартовых координатах вывести площади под трактрисой.
>> No.97836 Reply
>>97834
ККВМ >>84296
>> No.97844 Reply
>> No.97857 Reply
>>97836
неплохой был тред; пара содержательных срачей, кто-то, может быть, узнал что-то новое.

Примечательно, что сюда иногда заглядывают люди способные сказать что-то осмысленное, но, чтобы втянуть их в беседу, надо сначала спровоцировать картофанщиков, а разумные наблюдатели (не выдержав несовершенства мира) начинают их просвещать.

А ещё там была девочка-волшебница.
>> No.97861 Reply
>>97857
Ну-ка, няша, изложи свою точку зрения на то, почему 0.(9) = 1.
>> No.97862 Reply
>>97790
Интересно, спасибо. Свои замечания напишу позднее.
>> No.97885 Reply
File: ыфвыфв.png
Png, 1.01 KB, 152×76 - Click the image to expand
edit Find source with google Find source with iqdb
ыфвыфв.png
Что-то я в ступоре.
Не подскажете, каким образом проще всего достать х из скобок?
>> No.97886 Reply
>>97885
Добавлю.
Для этой функции нужно определить предел -> ∞, поэтому допустимо, чтобы в скобках осталось 4/x.
>> No.97887 Reply
>>97732
Грассман, Дедекинд и Пеано, похоже, рассуждали так же.
>> No.97889 Reply
>>97885
Бином Ньютона, лол.
>> No.97909 Reply
>> No.97910 Reply
File: 010100.pdf
Pdf, 0.36 KB, 595×841 - Click the image to get file
010100.pdf
>>97909
ГОСТ на математика.
>> No.97923 Reply
Привет анончики. Вот нужно найти обратную матрицу. И необходимо (не спрашивайте зачем, просто необходимо) переставить 2 столбца. Что при этом должно происходить с добавленной матрицей (которая из единичек справа, ну по которой находится обратная матрица). Ничего, да?
>> No.97932 Reply
>>97861
Более концептуально и теоретично скажу. Действительные числа по одному из этих дурацких определений - сечения множества рациональных чисел. Чтобы они были не равны между ними должно умещаться рациональное число. Вообще между любыми двумя действительными числами всегда бесконечно много как рациональных так и иррациональных. Между 0,(9) и 1 не вставить отличное от них число.
>>97885
Подумать пробовал? Срзли, ты что-то упускаешь, лол.
>> No.97933 Reply
>>97923
Это метод Гаусса? Так же вообще переставлять столбцы нельзя, как я помню.
>> No.97995 Reply
File: int.png
Png, 2.88 KB, 185×85 - Click the image to expand
edit Find source with google Find source with iqdb
int.png
Анон, прошу помощи. Имеется интеграл на пике. Он же решается с помощью замены переменной? Если да, то мы заменяем:
x^1/2=t
тогда t для x^1/6 в какой степени?
Начавший заранее готовится к сессии-кун
>> No.98000 Reply
>>97995
Подынтегральная функция - это рациональная функция от x^1/6. Т.к. x^1/3 - это x^1/6 в квадрате, а x^1/2 - это x^1/6 в кубе. x^1/6=t
>> No.98001 Reply
>>97995
> мы заменяем:
> x^1/2=t
Негодная замена. Бери x^(1/6). После выноса t^6 из под знака дифференциала получишь в числители t^6, и сократишь на t^2. Ну а дальше без меня разберёшься.
>> No.98010 Reply
>>98001
>>98000
Спасибо.
>> No.98069 Reply
Доброчан, посоветуй годных учебников для старшей школы по алгебре, особенно по производным и интегралам, подходящим для самообразования а то жизненно необходимо освоить программу двух классов за кратчайший срок, а по российским\советским учебникам учиться самому бесполезно. Можно на английском.
>> No.98070 Reply
>>98069
http://yadi.sk/d/9qMIaWMU0pBhV - Зорич
http://yadi.sk/d/UlT7nwzY0Vrck - Демидович
http://rghost.ru/43906548 - современная математика, тебе она скорее всего не понадобится, но имей в виду, что она есть
>> No.98083 Reply
Вопросы по прикладной статистике здесь задать будет можно?
>> No.98095 Reply
>>98083
Давай. Я постараюсь ответить.
>> No.98101 Reply
>>98070
спасибо, анон. хоть и не я спрашивал, но схоронил на будущее. у меня есть скромная просьба: нужна книга для самообучения алгебре и геометрии, т.е. нечто что позволит подойти к вышеприведённым учебникам Зорича и Демидовича полностью подготовленным.
>> No.98110 Reply
>>98101
Математика изучает абстрактные объекты. Всякий объект вводится с помощью определения. Изначально о рассматриваемом объекте мы знаем лишь то, что он обладает заявленными в определении свойствами. Эти первоначальные свойства мы называем аксиомами. На основе аксиом с помощью логики выводятся другие свойства - таким образом мы исследуем объект. Не всякий объект может быть признан существующим. Существующим мы считаем тот объект, свойства которого не противоречат друг другу; объект с противоречивыми свойствами мы считаем несуществующим.
Примеры математических объектов: множество, натуральные числа, геометрия Евклида, евклидово пространство, функция и т.п.

Суть математики - исследование математических объектов. Вычисления не являются сутью математики.

Математические объекты исследуются так: сперва мы делаем утверждение об объекте, а затем ищем доказательство этого утверждения. Доказательством утверждения мы называем логически правильное рассуждение, отправляющееся от определения объекта и заканчивающееся оным утверждением.
Если мы докажем, что исследуемый объект обладает неким свойством, то это ещё не даст нам повода утверждать, что объект не обладает противоположным свойством: исследуемый объект может быть противоречив. Однако очевидно, что если объект существует, то его свойства не противоречат друг другу. И потому как только мы докажем, что существующий объект обладает неким свойством, мы этим же самым докажем, что оный объект не обладает противоположным свойством.
Утверждение, что рассматриваемый нами объект обладает неким конкретным свойством, мы называем теоремой (если оно доказано) или гипотезой(если оно и не доказано, и не опровергнуто).
Леммой мы называем вспомогательное утверждение, нужное для доказательства теоремы и неинтересное само по себе.

Нужно заметить, что обывательские представления об аксиомах не совпадают с математическими. Для неспециалиста аксиома - это утверждение, которое не нуждается в доказательстве. Для математика аксиома - это часть определения; мысль о том, чтобы доказывать определение, несколько бредова.

Пример.
Полагаем натуральные числа и их умножение определёнными и известными.
Определение 1. Квадратом натурального числа n называют натуральное число `n*n. Будем обозначать квадрат n как n^2.
Определение 2. Число n - чётное, если существует такое число k, что n = 2*k.
Определение 3. Кракозяброй мы называем всякое натуральное число, квадрат которого - чётное число.

Теорема 1. Если число n - чётное, то оно - кракозябра.
Доказательство.
Пусть n - чётное число.
По определению 2 существует такое число k, что n=2*k.
Квадрат n есть n^2 = n*n = (2*k)*(2*k) = 2*(2*k*k) - по определению 1 и свойствам умножения натуральных чисел.
Обозначим 2*k*k как z.
Тогда n^2=2*z и по определению 2 n^2 есть чётное число.
Следовательно, по определению 3 число n есть кракозябра.
Теорема доказана.

Этот пример иллюстрирует типичное математическое рассуждение. Здесь мы ввели три объекта: квадрат натурального числа, чётное число и кракозябру. Мы задали их свойства и затем в ходе доказательства теоремы 1 опирались на них.
Мы открыли новое свойство чётных натуральных чисел: все они - кракозябры.
Нужно заметить, что мы не стали доказывать существования этих объектов, и потому научная ценность нашего рассуждения под вопросом.

Натуральные числа чаще всего определяются как множество, обладающее пятью свойствами, называемыми "аксиомы Пеано".
Множество - это абстрактный объект, обладающий свойствами, называемыми "аксиомы Цермело-Френкеля".

Важно различать сам абстрактный объект и его модель. Моделью абстрактного объекта x мы называем любой объект, обладающий перечисленными в определении x свойствами. То есть, например, комплексными числами может быть признан любой объект, в числе свойств которого содержатся, между прочим, аксиомы комплексных чисел. Это могут быть матрицы специального вида; это могут быть упорядоченые пары действительных чисел; это могут быть точки координатной плоскости и т.п. В некотором смысле всё перечисленное - один и тот же объект.

Важно также не давать слишком большую волю воображению. Мы работаем с абстрактными объектами, и нужно об этом помнить. Не стоит воображать алгебраическое кольцо как золотую безделушку: это безосновательное приписывание объекту лишних свойств. Об объекте мы знаем лишь то, что заявлено в определении, и то, что мы доказали. Больше мы не знаем ничего. На первых порах вообще можно считать математику игрой пустыми словами. Интуитивное понимание приходит с опытом.

В склонности работать с абстракциями есть минусы. Так, математический ум не может начать исследовать объекты, пока эти объекты не определены. Например, математик не начнёт рассуждать о душе, пока не получит её определения - поскольку не имеют свойств, из которых будет делать выводы. Однако в этой склонности есть и свои плюсы. Например, математик может начать читать книгу по алгебраической топологии, не изучая всю математику от Евклида до Клейна, поскольку все необходимые определения обязательно вводятся в самой книге. Лишь самые фундаментальные понятия вроде множества или топологического пространства авторы обычно не считают нужным определять.

Ближе к телу.
Для учебника Зорича не нужна никакая подготовка, кроме совсем тривиальной. Нужно владеть арифметикой и иметь небольшое представление о рациональных числах. Аксиомы рациональных чисел есть в вики и, как ни странно, у Фихтенгольца. Ещё нужна таблица умножения. Также нужно уметь складывать дроби, решать простенькие уравнения вроде x+2=5 и знать, как возводить число в рациональную степень.
Учебник же Демидовича написан не для математиков, а для... э-э... Лол, не знаю, для кого, неважно. Суть в том, что написан он преотвратно и может быть использован лишь в качестве справочника для людей, вынужденных учиться в рашковузе. Все используемые в учебнике Демидовича термины вроде синуса и тангенса нужно смотреть в википедии.
Стандартные школьные учебники с их "точками сгущения" читать вредно.

http://yadi.sk/d/vDaa8D_E0W-rL - Фихтенгольц. Можно читать только первый параграф, дальше читать нельзя.
http://yadi.sk/d/dPI2g4Ct0xFMk - учебник, официально предваряющий книгу Зорича. Именно он тебе нужен.
>> No.98136 Reply
Доброчан, помоги решить простую задачу, над которой бьюсь уже несколько дней, хотя раньше легко решал более сложные задачи.
f'(x)=g'(x)
f(x)=sin(2x-3)
g(x)=cos(2x-3)
Найти x.
Исписал много страниц, но в итоге захожу в тупик. Помоги!
>> No.98137 Reply
>>98136
f'(x)=2*cos(2x-3)
g'(x)=-2*sin(2x-3)
cos(2x-3)=-sin(2x-3)
cos(2x-3)+sin(2x-3)=0
sin(2x-3+(pi/4))=0
2x-3+(pi/4)=pi*n, n - целое число
x=(-(pi/4)+3+pi*n)/2
>> No.98145 Reply
File: 132305490460.jpg
Jpg, 300.46 KB, 800×1131 - Click the image to expand
edit Find source with google Find source with iqdb
132305490460.jpg
>>98137
Спасибо, няша! Держи няшку!
>> No.98172 Reply
>>98110
Для склонного к использованию логики читателя учебник Зорича будет отвратителен.
>> No.98182 Reply
>>98172
а почему ты свою перверсию называешь "склонностью к использованию логики" ?
>> No.98202 Reply
Правда ли, что парный линейный коэффициент корреляции (т.е. тот, что легко найти в том же excel и других программах) применим только в том случае, когда исследуемые выборки подчиняются нормальному распределению? Т.е. если оценить визуально гистограммы и там явная асимметрия, то нужно использовать непараметрические коэффициенты?
>> No.98214 Reply
File: af7e67e441fd08d31d76e0a0cfbff94d.pdf
Pdf, 0.91 KB, 595×878 - Click the image to get file
af7e67e441fd08d31d76e0a0cfbff94d.pdf
>>98202
Для нормальных сл. величин некоррелированнность (по Пирсону, т.е. "обычная линейная") тождественна независимости.
В общем случае из независимости следует некоррелированнность, но не наоборот.

Тестировать на равенство 0 коэффициента корреляции можно (см. например http://janda.org/c10/Lectures/topic06/L24-significanceR.htm)
НО такой тест очень сильно зависит от нормальности исходных распределений (Точнее, не совсем так. Подробнее см. приложенную статью)

Во многих случаях действительно лучше использовать непараметрические коэффициенты. Линейная корреляция была стандартом, когда все распределения для простоты считали нормальными. Сейчас теория развилась и так делать уже не модно.
>> No.98222 Reply
Никто не ответил мне, но я спрошу еще раз. Есть одна матрица. Квадратная. Невырожденная. И нужно найти обратную матрицу методом Гаусса, но с поиском не максимального элемента в столбце а с поиском максимального элемента во всей матрице. Как при этом искать обратную матрицу? Какие преобразования делать с добавленной (единичной) матрицей при перестановке столбцов?
>> No.98232 Reply
>>98222
Раньше было как: [M|E] -> [E|M^(-1)] : M*M^(-1) = E Теперь будет: [W|E'] -> [E'|G] : W*G = E' где W=M*P, E'=E*P, P - матрица перестановки
W*G = E' <=> M*P*G = E*P тогда
   E = M*P*G*(P^(-1)) = M*P*G*(PT), PT - транспонированная P

M*M^(-1) = M*P*G*(PT), домножим на M^(-1) раз она существует: M^(-1) = P*G*(PT) Т.е. ты делаешь как всегда свои перестановки как основной матрице так и к единичной, делаешь Гаусса, получаешь G, а замет получаешь M^(-1) = P*G*(PT) Надеюсь верно
>> No.98234 Reply
>>98222
Зачем тебе переставлять столбцы?
Берёшь максимальный элемент. Допустим он в i-й строке, в j-м столбце.
Переставляешь местами i, j строки. Зануляешь j-й столбец обычными преобразованиями (кроме диагонального элемента).
Дальше аналогично (ищешь максимальный везде, кроме j-й строки и столбца)
>> No.98245 Reply
Вечером я кину код, а вы скажите, что я делаю не так. У меня считает обратную матрицу но правильные элементы стоят не на своих местах. Вот я и думаю что где то ошибка.
>> No.98271 Reply
void solve(double a, double b, int n)
{
int i, j, k, im, jm;
double max;
for (i=0; i<n; i++)
{
im=i;
   jm=i;
   max=a[i*n+i];
   for (j=i; j<n; j++)
   for (k=i; k<n; k++)
   if (a[j*n+k]>max)
   {
   max=a[j*n+k];
   im=j;
   jm=k;
   }
   for (j=0; j<n; j++)
   {
   max=a[i*n+j];
   a[in+j]=a[imn+j];
   a[im*n+j]=max;
   }
   for (j=0; j<n; j++)
   {
	max=b[i*n+j];
   b[in+j]=b[imn+j];
   b[im*n+j]=max;
   }
   for (j=0; j<n; j++)
   {
   max=a[i+j*n];
   a[i+jn]=a[jn+jm];
   a[j*n+jm]=max;
   }

   for (j=0; j<n; j++)
   {
	max=b[i+j*n];
   b[i+jn]=b[jn+jm];
   b[j*n+jm]=max;
   }
   k=i*n;
   max=a[k+i];
for (j=n-1; j>=i; j--)
a[k+j]=a[k+j]/max;

for (j=n-1; j>=0; j--)
b[k+j]=b[k+j]/max;

for (j=i+1; j<n; j++)
{
   max=a[j*n+i];
   for (k=i; k<n; k++)
   {
   a[jn+k]=a[jn+k]-maxa[in+k];
   }
   for (k=0; k<n; k++)
   {
   b[jn+k]=b[jn+k]-maxb[in+k];
   }
  
}
}
for (i=n-2; i>=0; i--)
{
   for (j=i; j>=0; j--)
   {
   for (k=0; k<n; k++)
   {
   b[jn+k]=b[jn+k]-a[jn+i+1]b[(i+1)*n+k];
   a[jn+k]=a[jn+k]-a[jn+i+1]a[(i+1)*n+k];
   }
  
   }


}


Вот. Если кому не лень-посмотрите. Про вырожденные матрицы знаю, потом допишу. b-единичная матрица. a-исходная. n-размер.
>> No.98281 Reply
>>98271
Тут звёздочки портят разметку кода. Выложи хотя бы на pastebin что ли
>> No.98284 Reply
>>98271
В началах и концах жирного шрифта должны быть звездочки, умножение.
>> No.98296 Reply
>> No.98326 Reply
>>98296
Это что такое? Почему внутри цикла поиска максимума, а не снаружи?
 for (j=i; j<n; j++)
 {
   max=a[i*n+j];
   a[i*n+j]=a[im*n+j];
   a[im*n+j]=max;
 }
Дальше
 for (j=0; j<n; j++)
 {
   max=a[i+j*n];
   a[i+j*n]=a[j*n+jm];
   a[j*n+jm]=max;
 }
Переставлять столбцы так просто нельзя, т.к. ты при этом домножаешь справа на матрицу, а не слева.
Алгоритм же основан на том, что ты начинаешь с уравнения A * A^(-1) = I
И постепенно, домножая обе части слева на простые матрицы (перестановка строк, прибавление одной строки к другой и т.д.), "уничтожаешь" множитель A.
Я написал, как нужно делать: >>98234
>> No.98327 Reply
>>98326
Мне по заданию нужно методом гаусса. А это-с первой строки первого столбца.
Алсо проверил на работоспособность без перестановки столбцов. То есть искал максимальный элемент в строке. Мне выдало почти что правильную обратную матрицу, кроме единственного верхнего левого элемента. Он отличался. Не можете объяснить, где я налажал в коде начиная с for (i=n-2...) или мне в /s/?
>> No.98328 Reply
>>98327
> или мне в /s/
Я не знаю, я сам тут ньюфаг.

Напиши тогда точно, как звучит задание. потому что
> с первого столбца
> с поиском не максимального элемента в столбце а с поиском максимального элемента во всей матрице
вроде бы несовместимые вещи
>> No.98336 Reply
Метод Гаусса для нахождения обратной матрицы с выбором главного элемента во всей матрице.
>> No.98338 Reply
>>98327
Ты бы потрассировал, локализовал ошибку. Тебе надо учиться отлаживать код самостоятельно, а не выбрасывать его в тред целиком.
С:филонишь пожать
>> No.98358 Reply
File: lol-jackie-chan.jpg
Jpg, 43.20 KB, 387×512 - Click the image to expand
edit Find source with google Find source with iqdb
lol-jackie-chan.jpg
>>98338
> Тебе надо учиться
>> No.98404 Reply
Слава Лагранжу, разобрался с задачей. Там в конце просто поменял порядок вычитания и еще вычел в конце вне цикла один элемент. Теперь все работает даже для перестановки столбца и строки после некоторых танцев с бубном.
>> No.98411 Reply
Анон, а можешь составить список литературы для того, чтобы начать разбираться в математике и понимать, что происходит в ее современном мире? Просто, я так понимаю, программы мехмата, матфака ВШЭ и НМУ не соответствуют реальности, судя, хотя бы, по программам университетов лиги плюща.
>> No.98412 Reply
>>98411
> судя, хотя бы, по программам университетов лиги плюща
Ссылку в студию.
>> No.98414 Reply
>>98412
Я пришел спрашивать, а не вступать в полемику. Если анонимус способен составить такой список литературы - буду рад, если покажет, что я ошибаюсь, и программы названных мною вузов актуальны - буду рад, если даст ссылку на актуальную учебную программу по математике с учебными материалами - также буду рад.
>> No.98415 Reply
>>98414
И всё же дай ссылку.
>> No.98424 Reply
>>93583
> а также отвечаем на вопросы доброанонов.
Пожалуйста, покажите мне иррациональное число
>> No.98428 Reply
>>98424
Корень из двух.
>> No.98432 Reply
>>98411
> понимать, что происходит в ее современном мире
Для того, чтобы понять саму терминологию, т.е. язык, которым пользуется современная математика, нужно прочитать много книжек.
Потом нужно прочитать ещё больше, чтобы научиться говорить и понимать на это языке что-то осмысленное.

Поэтому запрашиваемый список литературы это список учебников по базовым предметам (и ещё лет пять свободного времени на их усвоение).
> ВШЭ и НМУ
Нужно разделять бакалавриат и аспирантские курсы.
Общематематическое образование в вшэ-нму довольно высокого качества (хотя, тут есть много тонкостей), а аспирантских курсов мало, потому что их некому слушать: старательные студенты уезжают в аспирантуру всё в ту же лигу плюща и другие места.
>> No.98433 Reply
Требую, чтобы следующий тред начинался с линкрилэйтеда или хотя бы содержал на него ссылку.
Но если кто-то может составить свой успешносписок лучше того, со тканями и мембранами, то эта будет хорошо, да.
http://dobrochan.org/rf/res/415483.xhtml
>> No.98434 Reply
>>98432
> хотя, тут есть много тонкостей
В чем они заключаются?
>> No.98443 Reply
>>98428
Извлеки его пожалуйста и покажи мне
>> No.98444 Reply
>>98443
Может, тебе ещё число i показать?
>> No.98445 Reply
Арнольд.-Геометрия-комплексных-чисел,-кватернионов.pdf
>> No.98446 Reply
>>98444
Нет. Мне нужно иррациональное число
>> No.98447 Reply
>>98446
Тебе его привели - корень из двух. Что ещё тебе нужно?
>> No.98450 Reply
>>98447
Нет. Вы мне привели 2 в степени 1/2. А мне нужно число, которое получится, если 2 возвести в степень 1/2. Покажите мне это число, пожалуйста
>> No.98451 Reply
>>98450
Ты думаешь, что число - это строка цифр? Лол. Число - нематериальная абстракция, оно задаётся перечислением его свойств и правилами обращения с ним. Обозначается разнообразными символами. Символ может быть составлен из нескольких значков. Символы "корень из двух", "2^(1/2)", "√2" обозначают (обычно) одно и то же число. Число можно обозначить строкой цифр, да. Но число - это, в общем случае, не строка цифр.

Известно, что нет никакого рационального числа, квадрат которого был бы равен корню из двух. Несоизмеримость диагонали квадрата с его стороной открыли ещё пифагорейцы. Чтобы извлечь корень из двух, построим новые числа, вещественные.

Пусть Q - множество всех рациональных чисел. Разобьём его на два множества Q' и Q'' так, чтобы
1)Q' и Q'' были бы непусты
2)всякое рациональное число относилось либо к Q', либо к Q''
3)всякое число из Q' было бы меньше всякого числа из Q''

Пару [Q',Q''] будем называть сечением (вещественным числом), Q' - нижним классом сечения, Q'' - верхним классом.

Построим некоторое сечение следующим образом.
К нижнему классу отнесём все рациональные числа, квадраты которых меньше 2. К верхнему классу отнесём все рациональные числа, квадраты которых больше двух.
Построенное вещественное число будет квадратным корнем из двух.

Более строгое изложение: http://www.mathesis.ru/books/dedekind4/mathesis_dedekind4.djvu
>> No.98453 Reply
>>98451
Важно понять, что каждое вещественное число образовано всеми рациональными числами, некоторым образом разбитыми надвое.

Если бы рациональные числа были бы конечным множеством {a,b,c,d}, то вещественных чисел было бы ровно три:
1: [{a}, {b,c,d}]
2: [{a,b}, {c,d}]
3: [{a,b,c}, {d}]
Но рациональные числа - бесконечное множество, поэтому сечения в области рациональных чисел выглядят сложнее.
>> No.98454 Reply
>>98451
Чем-то библию напоминает. Ты случайно не теолог?
>> No.98455 Reply
File: слоу2.gif
Gif, 98.17 KB, 500×364 - Click the image to expand
edit Find source with google Find source with iqdb
слоу2.gif
>>98454
Либо объяснись, либо проваливай из треда.
>> No.98456 Reply
>>98455
Чего объяснять? Число мне не показал, говоришь странные вещи множество, бесконечноссть, пытаешься меня ими запутать, злой ты математик. А теологию я привел в пример, т.к. там тоже есть Бог, но его существование не докажешь.
>> No.98457 Reply
>>98456
Я тебе даже сконструировал это число же. Что такое, по-твоему, число?
>> No.98458 Reply
>>98457
> Я тебе даже сконструировал это число же.
Я не видел. Кстати, разве не a^(1/2) = x, x^2 = a ? А то, что ты мне описал похоже на библейские сказки "о том, что нельзя увидеть и потрогать, но оно есть и имя ему Бог"
>> No.98459 Reply
>>98458
Что такое, по твоему мнению, число? Ответь.
>> No.98460 Reply
>>98459
Абстракция, которая помогает описывать количество.
>> No.98461 Reply
>>98460
Каким образом можно увидеть абстракцию?
>> No.98462 Reply
File: prikolnullnaa_fotopodborka_112.jpg
Jpg, 27.71 KB, 750×463 - Click the image to expand
edit Find source with google Find source with iqdb
prikolnullnaa_fotopodborka_112.jpg
>>98461
Интерпретировать ее в яблоки и оперировать ими же
>> No.98464 Reply
File: число.png
Png, 4.57 KB, 373×372 - Click the image to expand
edit Find source with google Find source with iqdb
число.png
>>98462
Глупо подменять абстракцию её интерпретацией. Это вводит в рассмотрение лишние свойства. Число 4, например, не имеет массы, а четыре яблока - имеют.

Интерпретируй корень из двух диагональю единичного квадрата, если хочешь.
>> No.98465 Reply
>>98464
> Число 4, например, не имеет массы, а четыре яблока - имеют.
А зачем мне рассматривать массу? Мне важно их количество.
Кстати, почему ты решил, что диагональ квадрата = √2? Аргументировать можешь?
>> No.98466 Reply
>>98465
> А зачем мне рассматривать массу? Мне важно их количество.
Ты работаешь не с числами, а с яблоками. Яблоки, в отличие от чисел, массивны. Ты не можешь от этого отказаться - если откажешься, то будешь делать суждения не о яблоках, а о каких-то абстрактных объектах.
> Аргументировать можешь?
По теореме Пифагора, квадрат гипотенузы равен сумме квадратов катетов. Пусть длина гипотенузы - x.
Тогда
x^2 = 1^2 + 1^2 = 1 + 1
x^2 = 2
Квадратным корнем из числа a называется такое число, квадрат которого равен a.
Таким образом, x - квадратный корень из 2.
>> No.98467 Reply
>>98466
Но если квадратный корень из 2 - число иррациональное, то как диагональ может иметь конечную длину?
>> No.98468 Reply
>>98467
Где ты видишь противоречие?
>> No.98469 Reply
>>98468
Не понимаю о чем ты. Я просто хочу посмотреть на иррациональное число, а мне его никто не может показать.
>> No.98470 Reply
>>98466
Перестань его кормить.
>>98434
Удваиваю.
>> No.98471 Reply
>>98469
Тебе описали сечение, являющееся иррациональным числом.
Впрочем, ты заявил, что хочешь посмотреть не на иррациональное число, а на интерпретацию иррационального числа. Тебе её показали - диагональ квадрата. Или сторона квадрата, зависит от точки зрения, сейчас расскажу подробнее.

Длина - это некое число, сопоставленное отрезку с помощью измерения. Это число показывает, сколько раз некий избранный единичный отрезок ("мера") уложится в измеряемом отрезке. Два отрезка называются соизмеримыми, если существует третий отрезок ("общая мера"), который укладывается натуральное число раз в каждом из двух отрезков.

Длины соизмеримых отрезков есть рациональные числа.

Диагональ квадрата несоизмерима с его стороной (пруф я уже дал). Это значит, что если мы изберём в качестве меры сторону квадрата, то длина диагонали не будет рациональным числом; если мы изберём в качестве меры диагональ, то длина стороны квадрата не будет рациональным числом.

Поэтому, чтобы всё-таки сопоставить каждому отрезку длину, мы и вводим новые числа - иррациональные. С помощью рациональных и иррациональных чисел каждый отрезок оказывается снабжённым длиной.
>> No.98472 Reply
File: 1366364252982.jpg
Jpg, 22.08 KB, 450×306 - Click the image to expand
edit Find source with google Find source with iqdb
1366364252982.jpg
>>98469
Ты либо тролль, либо феерически тупой. Убейся.
>> No.98473 Reply
>>98471
> Длины соизмеримых отрезков есть рациональные числа.
Допустим, есть избранный нами единичный отрезок E. Допустим, есть некий отрезок B, соизмеримый с ним. Отрезки соизмеримы, стало быть, есть общая мера.
Если общая мера укладывается в единичном отрезке e раз, а в отрезке B - b раз, то мы говорим, что длина отрезка B есть b/e.
Например, общая мера уложилась в B 8 раз, а в E - 7 раз. Тогда длина B есть 8/7.
>> No.98474 Reply
>>98472
Аргументируй, умняша
>> No.98492 Reply
Перекатываемся >>98491


Password:

[ /tv/ /rf/ /vg/ /a/ /b/ /u/ /bo/ /fur/ /to/ /dt/ /cp/ /oe/ /bg/ /ve/ /r/ /mad/ /d/ /mu/ /cr/ /di/ /sw/ /hr/ /wh/ /lor/ /s/ /hau/ /slow/ /gf/ /vn/ /w/ /ma/ /azu/ /wn/ ] [ Main | Settings | Bookmarks | Music Player ]